Discussion:
Definition für "zusammengesetze Funktion" und "unverzweigt verkettete Funktion" gesucht
(zu alt für eine Antwort)
IV
2018-07-29 13:17:14 UTC
Permalink
Könntet Ihr mir bitte Definitionen oder deren Quellen für folgende
Begriffe nennen? Ich als Laie bin doch dazu kaum in der Lage.
Es geht um Funktionen, die über Funktionsterme definiert sind (closed-form
functions).

Ich fange einfach mal an.

Definition I:
zusammengesetzte Funktion := eine Funktion, deren Funktionsterm aus
mindestens zwei Funktionstermen zusammengesetzt ist.

Beispiele für zusammengesetzte Funktionen:
Funktion F mit F(z) = f(f1(z))
Funktion F mit F(z) = f(f1(f2(z)))
Funktion F mit F(z) = f( f1(z), f2(z) )
Funktion F mit F(z) = f( f1( f11(z), f12(z) ), f2( f21(z)), f3(z) )
Oder muß ich immer schreiben "Funktion F mit z |-> F(z) = ..."?

univariate Funktion := Funktion einer Variablen
multivariate Funktion := Funktion mehrerer Variabler
Muß ich die Begriffe "Funktion einer Variablen" und "Funktion mehrerer
Variabler" noch definieren?

Definition II:
Seien n \in \mathbb{N}_0, und f_1, ..., f_n uni- oder multivariate
Funktionen.
zusammengesetzte Funktion := eine Funktion F mit F(z) = f_n(...(f_1(z))...)

Definition:
Seien n \in \mathbb{N}_0, und f_1, ..., f_n univariate Funktionen.
unverzweigt verkettete Funktion := eine Funktion F mit F(z) =
f_n(...(f_1(z))...)
Ich weiß nicht, wie ich H0Igers Einwurf "..., ist die Verkettung mitnichten
für Funktionen 'einer Variablen' definiert." zu verstehen habe.

Wie kann man die Definitionen und Begriffsbenennungen verbessern?

Vielen Dank für Eure Mitarbeit. Das Projekt soll der Allgemeinheit dienen,
nicht mir persönlich.
H0Iger SchuIz
2018-07-29 14:25:04 UTC
Permalink
Post by IV
zusammengesetzte Funktion := eine Funktion, deren Funktionsterm aus
mindestens zwei Funktionstermen zusammengesetzt ist.
Die Formulierung mit dem ":=" (was immer das bedeuten mag), ist schon
mal nix. Wolltest du nicht auf Deutsch schreiben? Von _dem_
Funktionsterm zu sprechen, dürfte wenig Sinn machen, so eindeutig dürfte
der nicht sein.

Gibt es nach der Definition denn auch eine Funktion, die nicht
zusammengesetzt ist?
Post by IV
Funktion F mit F(z) = f(f1(z))
Funktion F mit F(z) = f(f1(f2(z)))
Funktion F mit F(z) = f( f1(z), f2(z) )
Funktion F mit F(z) = f( f1( f11(z), f12(z) ), f2( f21(z)), f3(z) )
Oder muß ich immer schreiben "Funktion F mit z |-> F(z) = ..."?
univariate Funktion := Funktion einer Variablen
multivariate Funktion := Funktion mehrerer Variabler
Muß ich die Begriffe "Funktion einer Variablen" und "Funktion mehrerer
Variabler" noch definieren?
Ja.
Post by IV
Seien n \in \mathbb{N}_0, und f_1, ..., f_n uni- oder multivariate
Funktionen.
zusammengesetzte Funktion := eine Funktion F mit F(z) = f_n(...(f_1(z))...)
Zur Schreibweise, s.o. Inwiefern soll denn hier etwas anders als die
Komposition/Verkettung von Funktionen definiert werden?
Post by IV
Seien n \in \mathbb{N}_0, und f_1, ..., f_n univariate Funktionen.
unverzweigt verkettete Funktion := eine Funktion F mit F(z) =
f_n(...(f_1(z))...)
Ich weiß nicht, wie ich H0Igers Einwurf "..., ist die Verkettung mitnichten
für Funktionen 'einer Variablen' definiert." zu verstehen habe.
Schau dir die Definition für die
Verkettung/Komposition/Hintereinanderausführung an. Steht da irgendwo
etwas von "einer Variablen"?
Post by IV
Wie kann man die Definitionen und Begriffsbenennungen verbessern?
Erstmal solltem man sich klarmachen, was man eigentlich definieren will.
Dann sollte man bei den Formulierung nicht zu kanserig mit den Worten
sein. Ganze Sätze, gerne auch mehrere davon. Voraussetzungen benennen.
Formalisierte Schreibweisen, nur da, wo es passt. Insbesondere keine
Mischung von Sprache und Formalismus.
Post by IV
Vielen Dank für Eure Mitarbeit. Das Projekt soll der Allgemeinheit dienen,
nicht mir persönlich.
Ich dachte die Menschheit sei schon erlöst worden, weil im Nahen Osten
jemand zu Tode gefoltert wurde. Aber ein neuer Messias soll mir auch
recht sein.

hs
IV
2018-07-29 16:53:52 UTC
Permalink
Post by IV
zusammengesetzte Funktion := eine Funktion, deren Funktionsterm aus
mindestens zwei Funktionstermen zusammengesetzt ist.
Die Formulierung mit dem ":=" (was immer das bedeuten mag), ist schon mal
nix. Wolltest du nicht auf Deutsch schreiben?
(Ich scheue mich, Definitionen in ganzen Sätzen zu schreiben, denn das
"eine" in Definitionen scheint mir nicht eindeutig und deshalb eine Quelle
für Mißverständnisse zu sein.)
Von _dem_ Funktionsterm zu sprechen, dürfte wenig Sinn machen, so
eindeutig dürfte der nicht sein.
Das war auch mir bewußt.
Leider ist mir noch keine Definition des Begriffs "zusammengesetzte
Funktion" begegnet, obwohl er von Laien oft verwendet wird.
Aber wie immer danke auch für diese Anmerkung, denn:
Jetzt erst sehe ich, daß ich vermutlich zwei verschiedene Begriffe brauche:
"zusammengesetzte Funktion" wie oben definiert, der meint "über einen
zusammengesetzten Funktionsterm definierte Funktion" (für die mathematischen
Sätze in denen es eigentlich um Funktionsterme geht), und einen Begriff für
"aus gegebenen Standardfunktionen zusammengesetzte Funktion" (für die
zukünftigen mathematischen Sätze in denen es um Funktionenklassen geht).
Gibt es nach der Definition denn auch eine Funktion, die nicht
zusammengesetzt ist?
Nö. Das paßt ja zu meinen mathematischen Sätzen über zusammengesetzte
Funktionen, denn die gelten ja ebenfalls auch für 'triviale'
zusammengesetzte Funktionen.
Post by IV
Funktion F mit F(z) = f(f1(z))
Oder muß ich immer schreiben "Funktion F mit z |-> F(z) = ..."?
Muß ich die Begriffe "Funktion einer Variablen" und "Funktion mehrerer
Variabler" noch definieren?
Ja.
Och nö! Diese Begriffe werden in der Fachliteratur so häufig benutzt, aber
wohl nie exakt definiert.
Ich denke mal, man muß das über die Anzahl der Variablennamen im
Funktionsterm definieren.
Post by IV
Seien n \in \mathbb{N}_0, und f_1, ..., f_n uni- oder multivariate
Funktionen.
zusammengesetzte Funktion := eine Funktion F mit F(z) =
f_n(...(f_1(z))...)
Zur Schreibweise, s.o. Inwiefern soll denn hier etwas anders als die
Komposition/Verkettung von Funktionen definiert werden?
(Das hatte ich die ganze letzte Woche doch schon mehrmals geschrieben.)
jetzt mal formelmäßig - auf die Gefahr hin, daß Ihr wieder Inkorrektheiten
bemängeln könnt:

Definitionen:
Die Komposition zweier Funktionen f: Y --> Z und g: X --> Y ist die Funktion
h = f ∘ g: X --> Z, h(z) = f(g(z)).
oder:
Die Komposition zweier Funktionen f: Y --> Z und g: X --> Y0 \subseteq Y ist
die Funktion h = f ∘ g: X --> Z, h(z) = f(g(z)).
Beachte: Die Zielmenge der inneren Funktion ist eine T e i l m e n g e des
Definitionsbereiches der äußeren Funktion.

Ah, endlich habe ich etwas dazu gefunden:
Marti, Gröger: Grundkurs Mathematik für Ingenieure, Natur- und
Wirtschaftswissenschaftler
https://link.springer.com/chapter/10.1007/978-3-7908-2678-4_17
Definition:
Es seien f, g zwei Funktionen, so daß mindestens ein Element des
Wertebereiches von g im Definitionsbereich von f liegt: W_g \cap D_f \neq
\emptyset. Dann heißt die durch h(x) := f(g(x), D_h = {x: x \in D_g, g(x)
\in D_f} erklärte Funktion h Zusammensetzung von f mit g.
Beachte: Die Zielmenge der inneren Funktion muß keine Teilmenge des
Definitionsbereiches der äußeren Funktion sein, sondern es muß lediglich der
Durchschnitt beider Mengen ungleich \emptyset sein.
Post by IV
Post by IV
Seien n \in \mathbb{N}_0, und f_1, ..., f_n univariate Funktionen.
unverzweigt verkettete Funktion := eine Funktion F mit F(z) =
f_n(...(f_1(z))...)
Ich weiß nicht, wie ich H0Igers Einwurf "..., ist die Verkettung
mitnichten für Funktionen 'einer Variablen' definiert." zu verstehen
habe.
Schau dir die Definition für die
Verkettung/Komposition/Hintereinanderausführung an. Steht da irgendwo
etwas von "einer Variablen"?
Ja, denn: in meiner Definition oben sind es univariate Funktionen, die
miteinander verknüpft werden.
H0Iger SchuIz
2018-07-30 08:10:48 UTC
Permalink
Post by IV
Post by H0Iger SchuIz
Gibt es nach der Definition denn auch eine Funktion, die nicht
zusammengesetzt ist?
Nö.
Also ist der Begriff sinnlos. "Zusammengesetzte Funktion" ist dann ja
nur ein Synonym für "Funktion".
Post by IV
Das paßt ja zu meinen mathematischen Sätzen über zusammengesetzte
Funktionen, denn die gelten ja ebenfalls auch für 'triviale'
zusammengesetzte Funktionen.
Oh, Mann.
Post by IV
Post by H0Iger SchuIz
Post by IV
Funktion F mit F(z) = f(f1(z))
Oder muß ich immer schreiben "Funktion F mit z |-> F(z) = ..."?
Muß ich die Begriffe "Funktion einer Variablen" und "Funktion mehrerer
Variabler" noch definieren?
Ja.
Och nö! Diese Begriffe werden in der Fachliteratur so häufig benutzt, aber
wohl nie exakt definiert.
Denk' mal drüber nach, warum.
Post by IV
Post by H0Iger SchuIz
Post by IV
Seien n \in \mathbb{N}_0, und f_1, ..., f_n uni- oder multivariate
Funktionen.
zusammengesetzte Funktion := eine Funktion F mit F(z) =
f_n(...(f_1(z))...)
Zur Schreibweise, s.o. Inwiefern soll denn hier etwas anders als die
Komposition/Verkettung von Funktionen definiert werden?
(Das hatte ich die ganze letzte Woche doch schon mehrmals geschrieben.)
jetzt mal formelmäßig - auf die Gefahr hin, daß Ihr wieder Inkorrektheiten
Die Komposition zweier Funktionen f: Y --> Z und g: X --> Y ist die Funktion
h = f ? g: X --> Z, h(z) = f(g(z)).
Die Komposition zweier Funktionen f: Y --> Z und g: X --> Y0 \subseteq Y ist
die Funktion h = f ? g: X --> Z, h(z) = f(g(z)).
Beachte: Die Zielmenge der inneren Funktion ist eine T e i l m e n g e des
Definitionsbereiches der äußeren Funktion.
Marti, Gröger: Grundkurs Mathematik für Ingenieure, Natur- und
Wirtschaftswissenschaftler
https://link.springer.com/chapter/10.1007/978-3-7908-2678-4_17
Es seien f, g zwei Funktionen, so daß mindestens ein Element des
Wertebereiches von g im Definitionsbereich von f liegt: W_g \cap D_f \neq
\emptyset. Dann heißt die durch h(x) := f(g(x), D_h = {x: x \in D_g, g(x)
\in D_f} erklärte Funktion h Zusammensetzung von f mit g.
Das ist auf jeden Fall mal was anderes,a ls das, was du bisher
präsentiert hast. Was soll denn nun die Definition für "zusammengesetzt"
sein? Falls du diese hier übernehmen möchtest, möchte ich zur Vorsicht
raten: der Laie könnte von der Benennung der Definitionsbereiche
überfordert sein.

Ich seh' immer noch nicht, dass man diesen Begriff unbedingt braucht.
Letztendlich wird die innere Funktion so eingeschränkt, damit's für die
Hintereinanderausführung passt.
Post by IV
Post by H0Iger SchuIz
Post by IV
Post by IV
Seien n \in \mathbb{N}_0, und f_1, ..., f_n univariate Funktionen.
unverzweigt verkettete Funktion := eine Funktion F mit F(z) =
f_n(...(f_1(z))...)
Ich weiß nicht, wie ich H0Igers Einwurf "..., ist die Verkettung
mitnichten für Funktionen 'einer Variablen' definiert." zu verstehen
habe.
Schau dir die Definition für die
Verkettung/Komposition/Hintereinanderausführung an. Steht da irgendwo
etwas von "einer Variablen"?
Ja, denn: in meiner Definition oben sind es univariate Funktionen, die
miteinander verknüpft werden.
Mal abgesehen davon, dass ich noch nicht mitbekommen habe, dass der
Begriff "univariat" schon geklärt worden wäre, glaube ich, dass in diese
fall dein Definitionsversuch nicht mitspielt. Der Begriff der
Komposition ist schon belegt, du kannst ihn nicht einfach umdefinieren.

(Rein formal kannst du natürlich in deinen Arbeiten Begriffe definieren,
wie du möchtest. Zur Verständlichkeit trägt das aber nicht bei. Im
Gegenteil, Begriffe aus der mathematischen Folklore sollte man
übernehmen).
IV
2018-07-30 12:55:18 UTC
Permalink
Post by H0Iger SchuIz
Post by IV
Post by IV
Post by IV
Muß ich die Begriffe "Funktion einer Variablen" und "Funktion mehrerer
Variabler" noch definieren?
Ja.
Och nö! Diese Begriffe werden in der Fachliteratur so häufig benutzt,
aber wohl nie exakt definiert.
Denk' mal drüber nach, warum.
Hab' ich natürlich.
Post by H0Iger SchuIz
Post by IV
Post by IV
zusammengesetzte Funktionen
Marti, Gröger: Grundkurs Mathematik für Ingenieure, Natur- und
Wirtschaftswissenschaftler
https://link.springer.com/chapter/10.1007/978-3-7908-2678-4_17
Post by IV
Es seien f, g zwei Funktionen, so daß mindestens ein Element des
Wertebereiches von g im Definitionsbereich von f liegt: W_g \cap D_f \neq
\emptyset. Dann heißt die durch h(x) := f(g(x), D_h = {x: x \in D_g, g(x)
\in D_f} erklärte Funktion h Zusammensetzung von f mit g.
Ich seh' immer noch nicht, dass man diesen Begriff unbedingt braucht.
Ich brauche einen Begriff "zusammengesetzte Funktion", weil es in meinen
Sätzen um zusammengesetzte Funktionen gehen soll. Viele Funktionenklassen
lassen sich als zusammengesetzte Funktionen definieren, z. B. die
Elementaren Funktionen (Liouville / Ritt / Rosenlicht).
Post by H0Iger SchuIz
Ich seh' immer noch nicht, dass man diesen Begriff unbedingt braucht.
Letztendlich wird die innere Funktion so eingeschränkt, damit's für die
Hintereinanderausführung passt.
Der durch diese Definition erklärte Begriff "zusammengesetzte Funktion" ist
genau das was ich brauche und gemeint habe:
Der Anwender hat eine Funktion vorliegen, z. B. eine mit dem Funktionsterm
exp(sin(z)). Der Anwender betrachtet nicht DB und WB seiner Funktionen exp
und sin, kennt sie vielleicht auch gar nicht. Für die Anwendung benötigt er
nur DB oder BB dieser zusammengesetzten Funktion. Durch die
'Zusammensetzung' werden DB und WB von innerer und äußerer Funktion
eingeschränkt, diese sind ein Resultat, nicht zu Anfang gegebene Daten, der
Anwendung.
mathematische Anwendung: z. B. Bilden der Umkehrfunktion, z. B. für das
Auflösen einer gegebenen 'gewöhnlichen' Gleichung.
Post by H0Iger SchuIz
Post by IV
univariate Funktion := Funktion einer Variablen
multivariate Funktion := Funktion mehrerer Variabler
Muß ich die Begriffe "Funktion einer Variablen" und "Funktion mehrerer
Variabler" noch definieren?
Seien n \in \mathbb{N}_0, und f_1, ..., f_n uni- oder multivariate
Funktionen.
zusammengesetzte Funktion := eine Funktion F mit F(z) =
f_n(...(f_1(z))...)
Seien n \in \mathbb{N}_0, und f_1, ..., f_n univariate Funktionen.
unverzweigt verkettete Funktion := eine Funktion F mit F(z) =
f_n(...(f_1(z))...)
glaube ich, dass in diesem Fall dein Definitionsversuch nicht mitspielt.
Der Begriff der Komposition ist schon belegt, du kannst ihn nicht einfach
umdefinieren.
Die Funktion F ist eine 'n-stufige' Komposition. Für die Definition von F
kann ich aber nicht die übliche Definition der Komposition verwenden, weil
die anfangs gegebenen Funktionen, aus denen erst durch Einschränkung die
Gliedfunktionen der Komposition 'werden', 'beliebige' Funktionen sein
können.
H0Iger SchuIz
2018-07-30 14:10:05 UTC
Permalink
Post by IV
Ich brauche einen Begriff "zusammengesetzte Funktion", weil es in meinen
Sätzen um zusammengesetzte Funktionen gehen soll.
Soso. Deshalb war ja auch lange unklar, was das denn sein soll.
Schießlich geht es darum.
Post by IV
Viele Funktionenklassen
lassen sich als zusammengesetzte Funktionen definieren, z. B. die
Elementaren Funktionen (Liouville / Ritt / Rosenlicht).
Keine Ahnung, von welcher Definition von elementaren Funktionen du da
ausgehst. Aber wenn du uns eine zeigen würdest, in der die Klasse mir
Hilfe von Zusammensetzungen definiert ist. Wenn man schlicht über
Hintereinanderausführungen definiert, dürfte auch keine andere
Funktionsklasse bei herauskommen, oder?
Post by IV
Post by H0Iger SchuIz
Ich seh' immer noch nicht, dass man diesen Begriff unbedingt braucht.
Letztendlich wird die innere Funktion so eingeschränkt, damit's für die
Hintereinanderausführung passt.
Der durch diese Definition erklärte Begriff "zusammengesetzte Funktion" ist
Der Anwender hat eine Funktion vorliegen, z. B. eine mit dem Funktionsterm
exp(sin(z)). Der Anwender betrachtet nicht DB und WB seiner Funktionen exp
und sin, kennt sie vielleicht auch gar nicht.
Ich wüsste gern mal, was der fiktive Anwender eigentlich für ein Problem
hat. Er "hat" also eine Funktion "vorliegen", aber er weiß noch nicht
mal, was er da einsetzen kann und was dabei so 'raus kommen kann? Dann
kann er mit der Funktion wohl wenig anfangen. Funktionswerte wird er so
wohl kaum berechnen können.
Post by IV
Für die Anwendung benötigt er
nur DB oder BB dieser zusammengesetzten Funktion. Durch die
'Zusammensetzung' werden DB und WB von innerer und äußerer Funktion
eingeschränkt, diese sind ein Resultat, nicht zu Anfang gegebene Daten, der
Anwendung.
Eben. Eischränken muss man, damit's passt. Eh klar. Allerdings wird das
mit der Einschränkerei nichts, wenn man nicht weiß, welche Mange man
einschränken soll.
Post by IV
mathematische Anwendung: z. B. Bilden der Umkehrfunktion, z. B. für das
Auflösen einer gegebenen 'gewöhnlichen' Gleichung.
Macht man wohl nach all dem Eischränken. Ob das nun eine Zusammensetzung
oder die gute alte Hintereinanderausführung ist, dürfte für das Lösen
der Gleichung keinen Unterschied machen. Oder? Gegenbeispiel?
Post by IV
Post by H0Iger SchuIz
Post by IV
univariate Funktion := Funktion einer Variablen
multivariate Funktion := Funktion mehrerer Variabler
Muß ich die Begriffe "Funktion einer Variablen" und "Funktion mehrerer
Variabler" noch definieren?
Seien n \in \mathbb{N}_0, und f_1, ..., f_n uni- oder multivariate
Funktionen.
zusammengesetzte Funktion := eine Funktion F mit F(z) =
f_n(...(f_1(z))...)
Seien n \in \mathbb{N}_0, und f_1, ..., f_n univariate Funktionen.
unverzweigt verkettete Funktion := eine Funktion F mit F(z) =
f_n(...(f_1(z))...)
glaube ich, dass in diesem Fall dein Definitionsversuch nicht mitspielt.
Der Begriff der Komposition ist schon belegt, du kannst ihn nicht einfach
umdefinieren.
Die Funktion F ist eine 'n-stufige' Komposition.
Du meinst, weil sie durch Hintereinanderausführung von n Funktionen
entsteht? Das ist keine Eigenschaft der Funktion, sondern eine dieser
speziellen Darstellung durch eben jene n Funktionen.
Post by IV
Für die Definition von F
kann ich aber nicht die übliche Definition der Komposition verwenden, weil
die anfangs gegebenen Funktionen, aus denen erst durch Einschränkung die
Gliedfunktionen der Komposition 'werden', 'beliebige' Funktionen sein
können.
Verstehe ich nicht. Wenn du aber etwas anderes als die Komposition
verwenden möchtest, solltest du es auch anders nennen.

hs
IV
2018-07-30 15:37:35 UTC
Permalink
Post by H0Iger SchuIz
Viele Funktionenklassen lassen sich als zusammengesetzte Funktionen
definieren, z. B. die Elementaren Funktionen (Liouville / Ritt /
Rosenlicht).
Keine Ahnung, von welcher Definition von elementaren Funktionen du da
ausgehst. Aber wenn du uns eine zeigen würdest, in der die Klasse mit
Hilfe von Zusammensetzungen definiert ist. Wenn man schlicht über
Hintereinanderausführungen definiert, dürfte auch keine andere
Funktionsklasse bei herauskommen, oder?
Ich interpretiere "Hintereinanderausführung" gleich "Komposition".
[Ritt 1925]:
"The elementary functions are understood here to be those which are obtained
in a finite number of steps by performing algebraic operations and taking
exponentials and logarithms."
Es geht darin um das Anwenden von Funktionen auf Funktionen. Es sind
nirgends DB oder WB vorausgesetzt. Vorausgesetzt ist "nur", daß es sich um
Funktionen (bzw. Operationen) handelt.
Post by H0Iger SchuIz
Der Anwender hat eine Funktion vorliegen, z. B. eine mit dem
Funktionsterm exp(sin(z)). Der Anwender betrachtet nicht DB und WB seiner
Funktionen exp und sin, kennt sie vielleicht auch gar nicht.
Ich wüsste gern mal, was der fiktive Anwender eigentlich für ein Problem
hat. Er "hat" also eine Funktion "vorliegen", aber er weiß noch nicht mal,
was er da einsetzen kann und was dabei so 'raus kommen kann? Dann
kann er mit der Funktion wohl wenig anfangen. Funktionswerte wird er so wohl
kaum berechnen können.
Beispiel:

gegeben: der Funktionsterm x + sin(x)
gesucht:

Kann eine der durch den Funktionsterm x + exp(sin(x)) repräsentierten
bijektiven Funktionen eine Umkehrfunktion haben, die einen geschlossenen
Ausdruck als Funktionsterm hat?
Hat eine der durch den Funktionsterm x + exp(sin(x)) repräsentierten
bijektiven Funktionen eine Umkehrfunktion die einen geschlossenen Ausdruck
als Funktionsterm hat?
Welchen Funktionsterm hat die Umkehrfunktion einer gegebenen durch den
Funktionsterm x + sin(x) repräsentierten bijektiven Funktion?

Kann eine der durch den Funktionsterm x + exp(sin(x)) repräsentierten
Funktionen eine partielle (oder lokale) Umkehrfunktion haben, die einen
geschlossenen Ausdruck als Funktionsterm hat?
Hat eine der durch den Funktionsterm x + exp(sin(x)) repräsentierten
Funktionen eine partielle (oder lokale) Umkehrfunktion die einen
geschlossenen Ausdruck als Funktionsterm hat?
Welchen Funktionsterm hat die partielle (oder lokale) Umkehrfunktion einer
gegebenen durch den Funktionsterm x + sin(x) repräsentierten bijektiven
Funktion?

Welche Standardfunktionen muß ich in die Definition der Elementaren
Funktionen mit hineinnehmen, damit die oben gesuchten Funktionsterme
geschlossene Ausdrücke sind?

gegeben: die 'gewöhnliche Gleichung' x + exp(sin(x)) = 3
==> Die durch den Funktionsterm x + exp(sin(x)) repräsentierten Funktionen
sind gegeben. Durch Anwenden partieller Umkehrfunktionen dieser Funktionen
kann die Gleichung gelöst werden. Dann die Fragen oben.
Dann: Wenn ein geschlossener Ausdruck als Funktionsterm existiert,
dann kann die Gleichung durch Umformen aufgelöst werden, wenn nicht, dann
nicht. Oder man wendet meinen Satz 4 an und kriegt die Funktionsterme der
partiellen Umkehrfunktionen geliefert. In diese braucht man dann nur noch
die 3 an Stelle des x einsetzen.

Für die Elementaren Funktionen hat Ritt in [Ritt 1925] die Antwort gegeben:
Nur bijektive (= umkehrbare) elementare Funktionen die eine Verkettung
lediglich univariater Funktionen sind, sind elementar umkehrbar.
Ich will das erweitern auf alle Funktionenklassen, die nur nicht trivial und
trivial zusammengesetzte Funktionen enthalten.
Mit meinen Sätzen 4 und 5 folgt:
Jede bijektive elementare Funktion die eine Verkettung lediglich
univariater Funktionen ist, ist elementar umkehrbar. Mein Satz liefert die
"Struktur" des Funktionsterms und die Gliedfunktionen.
Jede bijektive elementare Funktion die eine Verkettung lediglich
univariater Funktionen ist, hat eine Umkehrfunktion der im Satz angegebenen
"Struktur" und hat die im Satz angegebene "Struktur".
Jede bijektive Funktion die eine Verkettung lediglich univariater
Funktionen ist, hat eine Umkehrfunktion, die aus einer Funktionenklasse
stammt, die die aus den partiellen Umkehrfunktionen der Gliedfunktionen
zusammengesetzten Funktionen enthält, z. B. ein enstprechender Körper.
Weiterhin folgen aus den Sätzen die entsprechenden Aussagen für die
partiellen Umkehrfunktionen.

Mit Ritts Satz und meinen Sätzen kann man sofort folgern:
Die Umkehrfunktionen der durch den Funktionsterm exp(sin(x))
repräsentierten bijektiven Funktionen haben den Funktionsterm sin(exp(x)).
Die partiellen Umkehrfunktionen der durch den Funktionsterm exp(sin(x))
repräsentierten Funktionen haben den Funktionsterm sin(exp(x)).
Mit Hilfe von Ritts Satz kommt man zu dem Schluß:
Die durch den Funktionsterm x + exp(sin(x)) repräsentierten bijektiven
Funktionen haben keine Umkehrfunktion in den Elementaren Funktionen.
Ich will das auf die partiellen Funktionen und auf andere bzw. ganz
allgemeine Funktionenklassen erweitern.
Post by H0Iger SchuIz
Für die Anwendung benötigt er
nur DB oder BB dieser zusammengesetzten Funktion. Durch die
'Zusammensetzung' werden DB und WB von innerer und äußerer Funktion
eingeschränkt, diese sind ein Resultat, nicht zu Anfang gegebene Daten, der
Anwendung.
Eben. Eischränken muss man, damit's passt. Eh klar. Allerdings wird das
mit der Einschränkerei nichts, wenn man nicht weiß, welche Mange man
einschränken soll.
Post by H0Iger SchuIz
mathematische Anwendung: z. B. Bilden der Umkehrfunktion, z. B. für das
Auflösen einer gegebenen 'gewöhnlichen' Gleichung.
Macht man wohl nach all dem Eischränken. Ob das nun eine Zusammensetzung
oder die gute alte Hintereinanderausführung ist, dürfte für das Lösen
der Gleichung keinen Unterschied machen. Oder? Gegenbeispiel?
Post by H0Iger SchuIz
Post by IV
univariate Funktion := Funktion einer Variablen
multivariate Funktion := Funktion mehrerer Variabler
Muß ich die Begriffe "Funktion einer Variablen" und "Funktion mehrerer
Variabler" noch definieren?
Seien n \in \mathbb{N}_0, und f_1, ..., f_n uni- oder multivariate
Funktionen.
zusammengesetzte Funktion := eine Funktion F mit F(z) =
f_n(...(f_1(z))...)
Seien n \in \mathbb{N}_0, und f_1, ..., f_n univariate Funktionen.
unverzweigt verkettete Funktion := eine Funktion F mit F(z) =
f_n(...(f_1(z))...)
glaube ich, dass in diesem Fall dein Definitionsversuch nicht mitspielt.
Der Begriff der Komposition ist schon belegt, du kannst ihn nicht einfach
umdefinieren.
Die Funktion F ist eine 'n-stufige' Komposition.
Du meinst, weil sie durch Hintereinanderausführung von n Funktionen
entsteht? Das ist keine Eigenschaft der Funktion, sondern eine dieser
speziellen Darstellung durch eben jene n Funktionen.
Post by H0Iger SchuIz
Für die Definition von F
kann ich aber nicht die übliche Definition der Komposition verwenden, weil
die anfangs gegebenen Funktionen, aus denen erst durch Einschränkung die
Gliedfunktionen der Komposition 'werden', 'beliebige' Funktionen sein
können.
Verstehe ich nicht. Wenn du aber etwas anderes als die Komposition
verwenden möchtest, solltest du es auch anders nennen.

hs
H0Iger SchuIz
2018-07-30 16:23:24 UTC
Permalink
Post by IV
Post by H0Iger SchuIz
Viele Funktionenklassen lassen sich als zusammengesetzte Funktionen
definieren, z. B. die Elementaren Funktionen (Liouville / Ritt /
Rosenlicht).
Keine Ahnung, von welcher Definition von elementaren Funktionen du da
ausgehst. Aber wenn du uns eine zeigen würdest, in der die Klasse mit
Hilfe von Zusammensetzungen definiert ist. Wenn man schlicht über
Hintereinanderausführungen definiert, dürfte auch keine andere
Funktionsklasse bei herauskommen, oder?
Ich interpretiere "Hintereinanderausführung" gleich "Komposition".
"The elementary functions are understood here to be those which are obtained
in a finite number of steps by performing algebraic operations and taking
exponentials and logarithms."
Es geht darin um das Anwenden von Funktionen auf Funktionen. Es sind
nirgends DB oder WB vorausgesetzt. Vorausgesetzt ist "nur", daß es sich um
Funktionen (bzw. Operationen) handelt.
Dieder Stil gefällt mir nicht. Warum möchtest du ihn dir zum Vorbild
machen.
Post by IV
Post by H0Iger SchuIz
Der Anwender hat eine Funktion vorliegen, z. B. eine mit dem
Funktionsterm exp(sin(z)). Der Anwender betrachtet nicht DB und WB seiner
Funktionen exp und sin, kennt sie vielleicht auch gar nicht.
Ich wüsste gern mal, was der fiktive Anwender eigentlich für ein Problem
hat. Er "hat" also eine Funktion "vorliegen", aber er weiß noch nicht mal,
was er da einsetzen kann und was dabei so 'raus kommen kann? Dann
kann er mit der Funktion wohl wenig anfangen. Funktionswerte wird er so wohl
kaum berechnen können.
gegeben: der Funktionsterm x + sin(x)
Kann eine der durch den Funktionsterm x + exp(sin(x)) repräsentierten
bijektiven Funktionen eine Umkehrfunktion haben, die einen geschlossenen
Ausdruck als Funktionsterm hat?
f: {0} -> {1}, x |-> x + exp(sin(x))

ist offensichtlich bijektiv und hat die Umkehrfunktion

f^-1: {1} -> {0}, x |-> 0
Post by IV
Hat eine der durch den Funktionsterm x + exp(sin(x)) repräsentierten
bijektiven Funktionen eine Umkehrfunktion die einen geschlossenen Ausdruck
als Funktionsterm hat?
S.o.
Post by IV
Welchen Funktionsterm hat die Umkehrfunktion einer gegebenen durch den
Funktionsterm x + sin(x) repräsentierten bijektiven Funktion?
0
Post by IV
Kann eine der durch den Funktionsterm x + exp(sin(x)) repräsentierten
Funktionen eine partielle (oder lokale) Umkehrfunktion haben, die einen
geschlossenen Ausdruck als Funktionsterm hat?
"Lokale" Umkehrfunktion macht nur Sinn, wenn man eine Topologie
(behelfsweise eine Metrik) voraussetzen kann. Das meinst du geht ohne
Angabe eines Definitions- und Wertebereichs?
Post by IV
Hat eine der durch den Funktionsterm x + exp(sin(x)) repräsentierten
Funktionen eine partielle (oder lokale) Umkehrfunktion die einen
geschlossenen Ausdruck als Funktionsterm hat?
Welchen Funktionsterm hat die partielle (oder lokale) Umkehrfunktion einer
gegebenen durch den Funktionsterm x + sin(x) repräsentierten bijektiven
Funktion?
Welche Standardfunktionen muß ich in die Definition der Elementaren
Funktionen mit hineinnehmen, damit die oben gesuchten Funktionsterme
geschlossene Ausdrücke sind?
Konstante Funktionen dürften schon drin sein.
Post by IV
gegeben: die 'gewöhnliche Gleichung' x + exp(sin(x)) = 3
==> Die durch den Funktionsterm x + exp(sin(x)) repräsentierten Funktionen
sind gegeben. Durch Anwenden partieller Umkehrfunktionen dieser Funktionen
kann die Gleichung gelöst werden. Dann die Fragen oben.
S.o. Der Funktionswert 3 kommt nicht vor. Die Gleichung hat keine
Lösung. Fertig.

Wenn man eine Gleichung lösen will, sollte man zumindest angeben, wo man
nach Lösungen sucht. Da macht es schon für wesentliche einfachere
Funktionen einen Unterschied, ob man von den komplexen oder den reellen
Zahlen ausgeht.
Post by IV
Dann: Wenn ein geschlossener Ausdruck als Funktionsterm existiert,
dann kann die Gleichung durch Umformen aufgelöst werden, wenn nicht, dann
nicht. Oder man wendet meinen Satz 4 an und kriegt die Funktionsterme der
partiellen Umkehrfunktionen geliefert. In diese braucht man dann nur noch
die 3 an Stelle des x einsetzen.
Vorrechnen!
Post by IV
Nur bijektive (= umkehrbare) elementare Funktionen die eine Verkettung
lediglich univariater Funktionen sind, sind elementar umkehrbar.
Sicher? Oder solche, die eine solche Darstellung haben?
Post by IV
Ich will das erweitern auf alle Funktionenklassen, die nur nicht trivial und
trivial zusammengesetzte Funktionen enthalten.
?
Post by IV
Jede bijektive elementare Funktion die eine Verkettung lediglich
univariater Funktionen ist, ist elementar umkehrbar. Mein Satz liefert die
"Struktur" des Funktionsterms und die Gliedfunktionen.
Jede bijektive elementare Funktion die eine Verkettung lediglich
univariater Funktionen ist, hat eine Umkehrfunktion der im Satz angegebenen
"Struktur" und hat die im Satz angegebene "Struktur".
Jede bijektive Funktion die eine Verkettung lediglich univariater
Funktionen ist, hat eine Umkehrfunktion, die aus einer Funktionenklasse
stammt, die die aus den partiellen Umkehrfunktionen der Gliedfunktionen
zusammengesetzten Funktionen enthält, z. B. ein enstprechender Körper.
Weiterhin folgen aus den Sätzen die entsprechenden Aussagen für die
partiellen Umkehrfunktionen.
Die Umkehrfunktionen der durch den Funktionsterm exp(sin(x))
repräsentierten bijektiven Funktionen haben den Funktionsterm sin(exp(x)).
Damit kann man jetzt genau was anfangen, wenn man nicht weiß, welche
Werte man für x einsetzen kann? Welche Funktionen mit diesem
Funktionsterm sind denn nun bijektiv?

Mal sehen: exp(sin(0)) = 1. Aber (Umkehrung): sin(exp(1)) = sin(e) \not=
0. Nee, mit dem Term stimmt etwas nicht.
Post by IV
Die partiellen Umkehrfunktionen der durch den Funktionsterm exp(sin(x))
repräsentierten Funktionen haben den Funktionsterm sin(exp(x)).
Die durch den Funktionsterm x + exp(sin(x)) repräsentierten bijektiven
Funktionen haben keine Umkehrfunktion in den Elementaren Funktionen.
Und was stimmt dann mit meinem Beispiel oben nicht?

Ende vom Lied: Wenn ma so gar nichts über den Definitions- und
Wertebereich aussagt, findet man zu jedem Funktionsterm eine
trivialerweise bijektive Funktion.
Post by IV
Ich will das auf die partiellen Funktionen und auf andere bzw. ganz
allgemeine Funktionenklassen erweitern.
Supi.

hs
IV
2018-07-30 17:48:13 UTC
Permalink
Post by IV
"The elementary functions are understood here to be those which are
obtained in a finite number of steps by performing algebraic operations
and taking exponentials and logarithms."
Es geht darin um das Anwenden von Funktionen auf Funktionen. Es sind
nirgends DB oder WB vorausgesetzt. Vorausgesetzt ist "nur", daß es sich
um Funktionen (bzw. Operationen) handelt.
Dieser Stil gefällt mir nicht. Warum möchtest du ihn dir zum Vorbild
machen?
Ich habe hier von Euch u. a. gelernt: Erstmal alles Benötigte mathematisch
korrekt definieren, dann jede Aussage mathematisch korrekt formulieren und
beweisen.
Ich sage: Und erst danach kann man sich auf das Niveau der potentiellen
Anwender einstellen.
Post by IV
Kann eine der durch den Funktionsterm x + exp(sin(x)) repräsentierten
bijektiven Funktionen eine Umkehrfunktion haben, die einen geschlossenen
Ausdruck als Funktionsterm hat?
f: {0} -> {1}, x |-> x + exp(sin(x))
ist offensichtlich bijektiv und hat die Umkehrfunktion
f^-1: {1} -> {0}, x |-> 0
Ich weiß.
Für Dich hatte ich wieder zu schnell formuliert - weil all das noch nicht
zum im Moment zu bearbeitenden Arbeitspaket gehört. Gemeint ist ein
Funktionsterm, der keine Konstante ist.
Post by IV
Kann eine der durch den Funktionsterm x + exp(sin(x)) repräsentierten
Funktionen eine partielle (oder lokale) Umkehrfunktion haben, die einen
geschlossenen Ausdruck als Funktionsterm hat?
"Lokale" Umkehrfunktion macht nur Sinn, wenn man eine Topologie
(behelfsweise eine Metrik) voraussetzen kann. Das meinst du geht ohne
Angabe eines Definitions- und Wertebereichs?
Nö. Hatte ich wohl auch nicht geschrieben.
Welche Standardfunktionen muß ich in die Definition der Elementaren
Funktionen mit hineinnehmen, damit die oben gesuchten Funktionsterme
geschlossene Ausdrücke sind?
Konstante Funktionen dürften schon drin sein.
Jede konstante Funktion ist eine algebraische Funktion, deren Anwendung eine
algebraische Operation. Deshalb ist jede konstante Funktion eine elementare
Funktion.
Gemeint war das allgemein gestellte Problem. Wenn man einen Funktionsterm x
exp(x) hat, muß man zum Körper der Elementaren Funktionen die
Lambert-W-Funktion adjungieren, damit die globale, partielle oder lokale
Umkehrfunktion in dieser Funktionenklasse liegt.
Post by IV
gegeben: die 'gewöhnliche Gleichung' x + exp(sin(x)) = 3
==> Die durch den Funktionsterm x + exp(sin(x)) repräsentierten
Funktionen sind gegeben. Durch Anwenden partieller Umkehrfunktionen
dieser Funktionen kann die Gleichung gelöst werden. Dann die Fragen oben.
S.o. Der Funktionswert 3 kommt nicht vor. Die Gleichung hat keine Lösung.
Fertig.
Ich weiß nicht was Du damit meinst.
Ich bekomme mit Hilfe von Ritts Satz und meiner Vermutung über Funktionen
algebraisch unabhängiger Funktionen:
Eine Funktion mit dem Funktionsterm x + exp(sin(x)) kann keine elementare
Funktion als Umkehrfunktion oder partielle Umkehrfunktion haben.
Die Gleichung hat die relle Lösung ca. 0.862648108496807.
Dann: Wenn ein geschlossener Ausdruck als Funktionsterm existiert, dann
kann die Gleichung durch Umformen aufgelöst werden, wenn nicht, dann
nicht. Oder man wendet meinen Satz 4 an und kriegt die Funktionsterme der
partiellen Umkehrfunktionen geliefert. In diese braucht man dann nur noch
die 3 an Stelle des x einsetzen.
Vorrechnen!
Erst im vorvorletzten Arbeitspaket.
Post by IV
Nur bijektive (= umkehrbare) elementare Funktionen die eine Verkettung
lediglich univariater Funktionen sind, sind elementar umkehrbar.
Sicher? Oder solche, die eine solche Darstellung haben?
Jede Funktion, die eine derartige Darstellung hat, ist eine derartige
Funktion, und jede derartige Funktion hat eine derartige Darstellung.
Post by IV
Ich will das erweitern auf alle Funktionenklassen, die nur nicht trivial
und trivial zusammengesetzte Funktionen enthalten.
?
Auf Funktionenklassen, die über zusammengesetzte Funktionen definiert sind.
Post by IV
Die Umkehrfunktionen der durch den Funktionsterm exp(sin(x))
repräsentierten bijektiven Funktionen haben den Funktionsterm sin(exp(x)).
Damit kann man jetzt genau was anfangen, wenn man nicht weiß, welche Werte
man für x einsetzen kann? Welche Funktionen mit diesem Funktionsterm sind
denn nun bijektiv?
Das ist hiieer nicht die Frage. Ich bin noch weit entfernt davon, eine v o
l l s t ä n d i g e Lösungstheorie für 'gewöhnliche' Gleichungen anbieten
zu können. In der Literatur habe ich eine solche auch noch nicht gefunden.
Mal sehen: exp(sin(0)) = 1. Aber (Umkehrung): sin(exp(1)) = sin(e) \not=
0. Nee, mit dem Term stimmt etwas nicht.
...
Und was stimmt dann mit meinem Beispiel oben nicht?
Ja, mit dem Term stimmt etwas nicht.
Mal sehen:
gegeben: die Gleichung exp(sin(0)) = 1
'Umkehrung' durch Gleichungsumformung oder bei komplexerem Funktionsterm der
'Gleichungsfunktion' einfacher durch direktes Bilden der partiellen
Umkehrfunktionen über meine Sätze, erst mit Hilfe von Ritts Satz und meinem
Satz kann man herleiten, daß das mit elementaren Funktionen bzw. elementaren
Operationen überhaupt möglich ist: 0 = arcsin(ln(1)). 0 = 0.
Post by IV
Die partiellen Umkehrfunktionen der durch den Funktionsterm exp(sin(x))
repräsentierten Funktionen haben den Funktionsterm sin(exp(x)).
Die durch den Funktionsterm x + exp(sin(x)) repräsentierten bijektiven
Funktionen haben keine Umkehrfunktion in den Elementaren Funktionen.
Ende vom Lied: Wenn man so gar nichts über den Definitions- und
Wertebereich aussagt, findet man zu jedem Funktionsterm eine
trivialerweise bijektive Funktion.
Auch darum geht es hier nicht.
Es geht z. B. darum,
- zu entscheiden, ob die globalen oder partiellen Umkehrfunktionen in einer
gegebenen Funktionenklasse liegen oder nicht,
- welche Funktionsterme in Frage kommen, und
- welche Funktion(en) zu den algebraischen (oder den elementaren) Funktionen
adjungiert werden müssen, damit die Funktionsterme der globalen oder
partiellen Umkehrfunktionen als geschlossener Ausdruck darstellbar sind.
Es geht um Aussagen über die Funktionsterme globaler und partieller (Hach,
schön, daß wir diesen Terminus herausarbeiten konnten.) Umkehrfunktionen
zusammengesetzter Funktionen. Ist die betrachtete 'unverzweigt verkettete'
Funktion nicht bijektiv, sind die hergeleiteten Funktionsterme nur
Funktionsterme der partiellen Umkehrfunktionen. Ist die betrachtete
'unverzweigt verkettete' Funktion bijektiv, ist zusätzlich auch der
Funktionsterm der (globalen) Umkehrfunktion herleitbar.
Versuche doch bitte nicht nur, in dem allem keinen Sinn zu sehen, sondern
versuche doch mal, in Ritts unbeachtetem Satz einen Sinn zu finden. Auch
mich hat das Wochen gekostet. Aber es hat sich gelohnt.
Zunächst einmal geht es hier allein um Funktionsterme. Und wenn man die für
die globale und alle partiellen Umkehrfunktionen hergeleiteten
Funktionsterme hat, dann kann man sie anwenden und die Definitionsbereiche
dieser globalen und partiellen Umkehrfunktion(en) heraussuchen.
Eine Gleichung wie x + exp(x) = c braucht man gar nicht erst versuchen,
allein mit elementaren Operationen (= elementaren Funktion) umstellen zu
wollen: das ergibt sich mit Hilfe von Ritts Satz. Und ich habe das Problem
des Lösens 'gewöhnlicher' (Kann man so sagen?) Gleichungen "übersetzt" in
das Problem des Anwendens der Umkehrfunktion. Denn das Ermitteln der
Funktionstermr der globalen und partiellen Umkehrfunktion(en) erfolgt
bekanntlich über das Lösen ihrer Definitionsgleichungen.
H0Iger SchuIz
2018-07-30 18:35:15 UTC
Permalink
Post by IV
Post by IV
"The elementary functions are understood here to be those which are
obtained in a finite number of steps by performing algebraic operations
and taking exponentials and logarithms."
Es geht darin um das Anwenden von Funktionen auf Funktionen. Es sind
nirgends DB oder WB vorausgesetzt. Vorausgesetzt ist "nur", daß es sich
um Funktionen (bzw. Operationen) handelt.
Dieser Stil gefällt mir nicht. Warum möchtest du ihn dir zum Vorbild
machen?
Ich habe hier von Euch u. a. gelernt: Erstmal alles Benötigte mathematisch
korrekt definieren, dann jede Aussage mathematisch korrekt formulieren und
beweisen.
Nein. Man kann, wenn überhaupt, nur die wahren Aussagen beweisen. Und
was hat diese Erklörung mit dem Stil von Ritt zunrun?
Post by IV
Ich sage: Und erst danach kann man sich auf das Niveau der potentiellen
Anwender einstellen.
Aha.
Post by IV
Post by IV
Kann eine der durch den Funktionsterm x + exp(sin(x)) repräsentierten
bijektiven Funktionen eine Umkehrfunktion haben, die einen geschlossenen
Ausdruck als Funktionsterm hat?
f: {0} -> {1}, x |-> x + exp(sin(x))
ist offensichtlich bijektiv und hat die Umkehrfunktion
f^-1: {1} -> {0}, x |-> 0
Ich weiß.
Für Dich hatte ich wieder zu schnell formuliert - weil all das noch nicht
zum im Moment zu bearbeitenden Arbeitspaket gehört. Gemeint ist ein
Funktionsterm, der keine Konstante ist.
x + exp(sin(x)) ist eine Konstante? Interessant.
Post by IV
Post by IV
Kann eine der durch den Funktionsterm x + exp(sin(x)) repräsentierten
Funktionen eine partielle (oder lokale) Umkehrfunktion haben, die einen
geschlossenen Ausdruck als Funktionsterm hat?
"Lokale" Umkehrfunktion macht nur Sinn, wenn man eine Topologie
(behelfsweise eine Metrik) voraussetzen kann. Das meinst du geht ohne
Angabe eines Definitions- und Wertebereichs?
Nö. Hatte ich wohl auch nicht geschrieben.
Eben. Dann macht "lokale" Umkejrfunktion aber keinen Sinn.
Post by IV
Welche Standardfunktionen muß ich in die Definition der Elementaren
Funktionen mit hineinnehmen, damit die oben gesuchten Funktionsterme
geschlossene Ausdrücke sind?
Konstante Funktionen dürften schon drin sein.
Jede konstante Funktion ist eine algebraische Funktion, deren Anwendung eine
algebraische Operation. Deshalb ist jede konstante Funktion eine elementare
Funktion.
Gemeint war das allgemein gestellte Problem. Wenn man einen Funktionsterm x
exp(x) hat, muß man zum Körper der Elementaren Funktionen die
Lambert-W-Funktion adjungieren, damit die globale, partielle oder lokale
Umkehrfunktion in dieser Funktionenklasse liegt.
Was ist daran "allgemein" gestellt? Und inwiefern war das mit dem
Beispiel gemeint. Mir dünkt, er springt ein wenig in den Themen.
Post by IV
Post by IV
gegeben: die 'gewöhnliche Gleichung' x + exp(sin(x)) = 3
==> Die durch den Funktionsterm x + exp(sin(x)) repräsentierten
Funktionen sind gegeben. Durch Anwenden partieller Umkehrfunktionen
dieser Funktionen kann die Gleichung gelöst werden. Dann die Fragen oben.
S.o. Der Funktionswert 3 kommt nicht vor. Die Gleichung hat keine Lösung.
Fertig.
Ich weiß nicht was Du damit meinst.
Damit meine ich, dass für die Funktion

f: {0} -> {1}, x |-> x + exp(sin(x))

die Gleichung f(x) = 3 keine Lösung hat.
Post by IV
Ich bekomme mit Hilfe von Ritts Satz und meiner Vermutung über Funktionen
Eine Funktion mit dem Funktionsterm x + exp(sin(x)) kann keine elementare
Funktion als Umkehrfunktion oder partielle Umkehrfunktion haben.
Die Gleichung hat die relle Lösung ca. 0.862648108496807.
Ui, das sind aber viele Nachkommastellen. Davon muss man aj immer schön
viele aufschreiben. Diese Zahl liegt ja nicht im Definitionsbereich.
Post by IV
Dann: Wenn ein geschlossener Ausdruck als Funktionsterm existiert, dann
kann die Gleichung durch Umformen aufgelöst werden, wenn nicht, dann
nicht. Oder man wendet meinen Satz 4 an und kriegt die Funktionsterme der
partiellen Umkehrfunktionen geliefert. In diese braucht man dann nur noch
die 3 an Stelle des x einsetzen.
Vorrechnen!
Erst im vorvorletzten Arbeitspaket.
Also kann man die Gleichung nun durch Anwenden des Satzes lösen oder
nicht? Butter bei die Fische.
Post by IV
Post by IV
Nur bijektive (= umkehrbare) elementare Funktionen die eine Verkettung
lediglich univariater Funktionen sind, sind elementar umkehrbar.
Sicher? Oder solche, die eine solche Darstellung haben?
Jede Funktion, die eine derartige Darstellung hat, ist eine derartige
Funktion, und jede derartige Funktion hat eine derartige Darstellung.
Aha. Wie stellt amn denn fest, ob eine Funktion eine solche Darstellung
hat oder nicht?
Post by IV
Post by IV
Ich will das erweitern auf alle Funktionenklassen, die nur nicht trivial
und trivial zusammengesetzte Funktionen enthalten.
?
Auf Funktionenklassen, die über zusammengesetzte Funktionen definiert sind.
Auf Toast.
Post by IV
Post by IV
Die Umkehrfunktionen der durch den Funktionsterm exp(sin(x))
repräsentierten bijektiven Funktionen haben den Funktionsterm sin(exp(x)).
Damit kann man jetzt genau was anfangen, wenn man nicht weiß, welche Werte
man für x einsetzen kann? Welche Funktionen mit diesem Funktionsterm sind
denn nun bijektiv?
Das ist hiieer nicht die Frage.
Doch. Diese Frage habe ich gestellt. Das finde ich interessant.
Post by IV
Ich bin noch weit entfernt davon, eine v o
l l s t ä n d i g e Lösungstheorie für 'gewöhnliche' Gleichungen anbieten
zu können. In der Literatur habe ich eine solche auch noch nicht gefunden.
Warum bloß?
Post by IV
Mal sehen: exp(sin(0)) = 1. Aber (Umkehrung): sin(exp(1)) = sin(e) \not=
0. Nee, mit dem Term stimmt etwas nicht.
...
Und was stimmt dann mit meinem Beispiel oben nicht?
Ja, mit dem Term stimmt etwas nicht.
gegeben: die Gleichung exp(sin(0)) = 1
'Umkehrung' durch Gleichungsumformung oder bei komplexerem Funktionsterm der
'Gleichungsfunktion' einfacher durch direktes Bilden der partiellen
Umkehrfunktionen über meine Sätze, erst mit Hilfe von Ritts Satz und meinem
Satz kann man herleiten, daß das mit elementaren Funktionen bzw. elementaren
Operationen überhaupt möglich ist: 0 = arcsin(ln(1)). 0 = 0.
Ah, das ist aber ein ganz anderer Term als eben. Da haben wir doch wohl
keinen Fehler gemacht?

Wie sieht's denn mit exp(sin(4pi)) = 1 aus. Kann man das auch umkehren?
Post by IV
Post by IV
Die partiellen Umkehrfunktionen der durch den Funktionsterm exp(sin(x))
repräsentierten Funktionen haben den Funktionsterm sin(exp(x)).
Die durch den Funktionsterm x + exp(sin(x)) repräsentierten bijektiven
Funktionen haben keine Umkehrfunktion in den Elementaren Funktionen.
Ende vom Lied: Wenn man so gar nichts über den Definitions- und
Wertebereich aussagt, findet man zu jedem Funktionsterm eine
trivialerweise bijektive Funktion.
Auch darum geht es hier nicht.
Du kannst dieses Beispiel gerne ignorieren und später darüber stolpern.
Post by IV
Es geht z. B. darum,
- zu entscheiden, ob die globalen oder partiellen Umkehrfunktionen in einer
gegebenen Funktionenklasse liegen oder nicht,
- welche Funktionsterme in Frage kommen, und
- welche Funktion(en) zu den algebraischen (oder den elementaren) Funktionen
adjungiert werden müssen, damit die Funktionsterme der globalen oder
partiellen Umkehrfunktionen als geschlossener Ausdruck darstellbar sind.
Es geht um Aussagen über die Funktionsterme globaler und partieller (Hach,
schön, daß wir diesen Terminus herausarbeiten konnten.
Konnten "wir" das? Was ist denn nun eine partielle Umkehrfunktion?
Post by IV
) Umkehrfunktionen
zusammengesetzter Funktionen. Ist die betrachtete 'unverzweigt verkettete'
Funktion nicht bijektiv, sind die hergeleiteten Funktionsterme nur
Funktionsterme der partiellen Umkehrfunktionen. Ist die betrachtete
'unverzweigt verkettete' Funktion bijektiv, ist zusätzlich auch der
Funktionsterm der (globalen) Umkehrfunktion herleitbar.
Versuche doch bitte nicht nur, in dem allem keinen Sinn zu sehen,
Ich hinterfrage, nicht mehr, nicht weniger. Wenn dich meine Hinweise
nicht interessieren (oder su sie nicht verstehst), dann ist dann eben
so. Ich halte es jedenfalls für gequirlten Quatsch Funktionen ohne
Definitionsbereich zu betrachten. Ich glaube auch nicht, dass Ritt das
im Sinn gehabt hat, nur weil er das etwas lax aufgeschrieben hat.
Post by IV
sondern
versuche doch mal, in Ritts unbeachtetem Satz einen Sinn zu finden.
Ja, dadurch wird die Menschheit erlöst.
Post by IV
Auch
mich hat das Wochen gekostet. Aber es hat sich gelohnt.
Zunächst einmal geht es hier allein um Funktionsterme. Und wenn man die für
die globale und alle partiellen Umkehrfunktionen hergeleiteten
Funktionsterme hat, dann kann man sie anwenden und die Definitionsbereiche
dieser globalen und partiellen Umkehrfunktion(en) heraussuchen.
Eine Gleichung wie x + exp(x) = c braucht man gar nicht erst versuchen,
allein mit elementaren Operationen (= elementaren Funktion) umstellen zu
wollen: das ergibt sich mit Hilfe von Ritts Satz.
Und ich habe das Problem
des Lösens 'gewöhnlicher' (Kann man so sagen?) Gleichungen "übersetzt" in
das Problem des Anwendens der Umkehrfunktion. Denn das Ermitteln der
Funktionstermr der globalen und partiellen Umkehrfunktion(en) erfolgt
bekanntlich über das Lösen ihrer Definitionsgleichungen.
Prima. Dann ist ja alles in Ordnung. Vergess' nicht, ein paar Beispiele
in deine Veröffentlichungen zu schreiben, die kucke ich mir gerne an.

hs
IV
2018-07-30 21:52:40 UTC
Permalink
Post by H0Iger SchuIz
Post by IV
Ich habe hier von Euch u. a. gelernt: Erstmal alles Benötigte
mathematisch korrekt definieren, dann jede Aussage mathematisch korrekt
formulieren und beweisen.
Nein. Man kann, wenn überhaupt, nur die wahren Aussagen beweisen.
Dein "Nein" scheint mir zu streng zu sein. Unwahre Aussagen und Mengen, für
die kein Element existiert, benötige ich doch nicht.
Post by H0Iger SchuIz
Und was hat diese Erklärung mit dem Stil von Ritt zu tun?
Erstens: Bei Liouville und Ritt geht Alles ohne explizit genannte
Definitions- und Wertebereiche.
Zweitens: Erst nachdem ich das Benötigte korrekt definiert, formuliert und
bewiesen habe, kann ich mich auf das spärliche Formulierungsniveau von Ritt
begeben - im Sinne der potentiellen Anwender.
Post by H0Iger SchuIz
Post by IV
Post by H0Iger SchuIz
Post by IV
Kann eine der durch den Funktionsterm x + exp(sin(x)) repräsentierten
bijektiven Funktionen eine Umkehrfunktion haben, die einen
geschlossenen Ausdruck als Funktionsterm hat?
f: {0} -> {1}, x |-> x + exp(sin(x))
ist offensichtlich bijektiv und hat die Umkehrfunktion
f^-1: {1} -> {0}, x |-> 0
Ich weiß.
Für Dich hatte ich wieder zu schnell formuliert - weil all das noch nicht
zum im Moment zu bearbeitenden Arbeitspaket gehört. Gemeint ist ein
Funktionsterm, der keine Konstante ist.
x + exp(sin(x)) ist eine Konstante? Interessant.
In x |-> 0 ist der Funktionsterm, 0, eine Konstante.
Post by H0Iger SchuIz
Post by IV
Post by H0Iger SchuIz
Post by IV
gegeben: die 'gewöhnliche Gleichung' x + exp(sin(x)) = 3
==> Die durch den Funktionsterm x + exp(sin(x)) repräsentierten
Funktionen sind gegeben. Durch Anwenden partieller Umkehrfunktionen
dieser Funktionen kann die Gleichung gelöst werden. Dann die Fragen oben.
S.o. Der Funktionswert 3 kommt nicht vor. Die Gleichung hat keine Lösung.
Fertig.
Ich weiß nicht was Du damit meinst.
Damit meine ich, dass für die Funktion
f: {0} -> {1}, x |-> x + exp(sin(x))
die Gleichung f(x) = 3 keine Lösung hat.
Ich sehe nicht, was Deine Funktion mit der von mir gegebenen Gleichung zu
tun hat. Implizite Antworten verstehe ich nicht.
Post by H0Iger SchuIz
Post by IV
Post by H0Iger SchuIz
Post by IV
Dann: Wenn ein geschlossener Ausdruck als Funktionsterm existiert, dann
kann die Gleichung durch Umformen aufgelöst werden, wenn nicht, dann
nicht. Oder man wendet meinen Satz 4 an und kriegt die Funktionsterme der
partiellen Umkehrfunktionen geliefert. In diese braucht man dann nur
noch die 3 an Stelle des x einsetzen.
Vorrechnen!
Erst im vorvorletzten Arbeitspaket.
Also kann man die Gleichung nun durch Anwenden des Satzes lösen oder
nicht? Butter bei die Fische.
Meine bisherigen Euch vorliegenden Satz-Entwürfe sagen etwas über globale
und partielle Umkehrfunktionen, nichts über Gleichungen.
Meine Gleichung x + exp(sin(x)) = 3?
Wenn nichts weiter über die DB und WB der 'Gliedfunktionen' und der
'Gleichungsfunktion' gegeben ist, muß man die DB und WB von exp und sin
nehmen.
Mit Ritts Satz und einem meiner weiteren Satz-Entwürfe folgt, daß x +
exp(sin(x)) nicht elementar umkehrbar ist. Da wir im Moment keine andere
Funktionenklasse wissen, durch die x + exp(sin(x)) in Form eines
Funktionsterms einer 'unverzweigt verketteten' Funktion darstellbar ist,
bleibt nur die numerische Lösung.
Post by H0Iger SchuIz
Post by IV
Post by H0Iger SchuIz
Post by IV
Nur bijektive (= umkehrbare) elementare Funktionen die eine Verkettung
lediglich univariater Funktionen sind, sind elementar umkehrbar.
Sicher? Oder solche, die eine solche Darstellung haben?
Jede Funktion, die eine derartige Darstellung hat, ist eine derartige
Funktion, und jede derartige Funktion hat eine derartige Darstellung.
Aha. Wie stellt man denn fest, ob eine Funktion eine solche Darstellung
hat oder nicht?
Das hatten wir in einem anderen Thread schon mal.
Ritt schreibt dazu: "It remains to develop a method for recognizing whether
a given elementary function can be reduced to the above form for F(z). How
to test fairly simple functions will be evident from the details of our
proofs. For the immediate present, we let the general question stand."
Meine Vermutung ist, daß eine Funktion, deren 'Verkettungsdarstellung' eine
Funktion algebraisch unabhängiger Funktionen enthält, prinzipiell nicht in
dem zugrundegelegten Körper umkehrbar ist.
Post by H0Iger SchuIz
Post by IV
Auf Funktionenklassen, die über zusammengesetzte Funktionen definiert sind.
Auf Toast.
Ja, Du darfst den Toast dann aussprechen.
Post by H0Iger SchuIz
Post by IV
Post by H0Iger SchuIz
Post by IV
Die Umkehrfunktionen der durch den Funktionsterm exp(sin(x))
repräsentierten bijektiven Funktionen haben den Funktionsterm sin(exp(x)).
Damit kann man jetzt genau was anfangen, wenn man nicht weiß, welche
Werte man für x einsetzen kann? Welche Funktionen mit diesem
Funktionsterm sind denn nun bijektiv?
Das ist hiieer nicht die Frage.
Doch. Diese Frage habe ich gestellt. Das finde ich interessant.
Ich habe darauf keine Antwort. Vielleicht fällt ja eine Antwort bei der
Bearbeitung der anderen Teilaufgaben des Projekts ab.
Ja, hast Du denn eine Idee?
Post by H0Iger SchuIz
Post by IV
Ich bin noch weit entfernt davon, eine v o l l s t ä n d i g e
Lösungstheorie für 'gewöhnliche' Gleichungen anbieten zu können. In der
Literatur habe ich eine solche auch noch nicht gefunden.
Warum bloß?
Weil nicht jeder Liouville, Ritt, Rosenlicht, Khovanskii und Burda kennt.
Post by H0Iger SchuIz
Post by IV
Post by H0Iger SchuIz
Post by IV
Mal sehen: exp(sin(0)) = 1. Aber (Umkehrung): sin(exp(1)) = sin(e) \not=
0. Nee, mit dem Term stimmt etwas nicht.
...
Und was stimmt dann mit meinem Beispiel oben nicht?
Ja, mit dem Term stimmt etwas nicht.
gegeben: die Gleichung exp(sin(0)) = 1
'Umkehrung' durch Gleichungsumformung oder bei komplexerem Funktionsterm
der 'Gleichungsfunktion' einfacher durch direktes Bilden der partiellen
Umkehrfunktionen über meine Sätze, erst mit Hilfe von Ritts Satz und
meinem Satz kann man herleiten, daß das mit elementaren Funktionen bzw.
elementaren Operationen überhaupt möglich ist: 0 = arcsin(ln(1)). 0 = 0.
Ah, das ist aber ein ganz anderer Term als eben. Da haben wir doch wohl
keinen Fehler gemacht?
Wir nicht. Du. Die Umkehrung erfolgt (wie beim Gleichunglösen auch) durch
Anwenden der partiellen Umkehrfunktionen der Gliedfunktionen in umgekehrter
Reihenfolge.
Post by H0Iger SchuIz
Wie sieht's denn mit exp(sin(4pi)) = 1 aus. Kann man das auch umkehren?
Also eigentlich ist mein Thema "globale und partielle Umkehrfunktionen
zusammengesetzter Funktionen einer Variablen, und 'gewöhnliche' Gleichungen
einer Variablen".
Ist die Frage, wonach umgestellt werden soll: nach sin(4pi), 4, oder pi.
Man sieht sofort: Für jeden der eben genannten Parameter ist die
Darstellung eine 'unverzweigt verkettete Funktion', also in der Form von
Ritts Satz und meinen Sätzen.
Daher weiß man sofort, daß man hier mit elementaren partiellen
Umkehrfunktionen (elementaren Operationen) auskommt.
exp(sin(4 pi)) = 1
4 pi = Arcsin(Ln(1))
4 pi = arcsin(ln(1)) + 4 pi
4 pi = 4 pi
Post by H0Iger SchuIz
Post by IV
partielle Umkehrfunktionen (Hach, schön, daß wir diesen Terminus
herausarbeiten konnten.)
Konnten "wir" das? Was ist denn nun eine partielle Umkehrfunktion?
Ja, mit Eurer Hilfe habe ich es gefunden.
https://en.wikipedia.org/wiki/Inverse_function#Partial_inverses
Post by H0Iger SchuIz
Post by IV
Versuche doch bitte nicht nur, in dem allem keinen Sinn zu sehen,
Ich hinterfrage, nicht mehr, nicht weniger. Wenn dich meine Hinweise nicht
interessieren (oder du sie nicht verstehst), dann ist das eben so. Ich
halte es jedenfalls für gequirlten Quatsch, Funktionen ohne
Definitionsbereich zu betrachten.
Ich betrachte auch nicht Funktionen, sondern
'Gruppen'/'Klassen'/'Mengen'/'Zoos' von Funktionen mit gleichem
Funktionsterm oder gleicher Form des Funktionsterm.
Post by H0Iger SchuIz
Ich glaube auch nicht, dass Ritt das im Sinn gehabt hat, nur weil er das
etwas lax aufgeschrieben hat.
Na, er hat immerhin noch geschrieben "die Funktion phi(z)".
Post by H0Iger SchuIz
Prima. Dann ist ja alles in Ordnung. Vergess' nicht, ein paar Beispiele in
deine Veröffentlichungen zu schreiben, die kucke ich mir gerne an.
Mathematische Veröffentlichungen zeichnen sich zu meinem Leidwesen leider
dadurch aus, daß sie selten Beispiele enthalten.
IV
2018-07-31 10:51:07 UTC
Permalink
Post by IV
exp(sin(4 pi)) = 1
4 pi = Arcsin(Ln(1))
4 pi = arcsin(ln(1)) + 4 pi
4 pi = 4 pi
exp(sin(4 pi)) = 1
Entschuldigung. An dieser Stelle bieten sich die Umkehrrelationen an:
4 pi = Arcsin(Ln(1))
Ich werde also auch den bekannten Satz über die "Zusammensetzung" der
Umkehrrelation 'zusammengesetzter' Relationen mit aufnehmen müssen.
Und erst jetzt kommen DB und WB ins Spiel, nämlich die der partiellen
Umkehrfunktionen ('Zweige' der Umkehrrelation), und auch nur die, die
überhaupt relevant sind für die Gleichung.
4 pi = arcsin(ln(1)) + 4 pi
4 pi = 4 pi
H0Iger SchuIz
2018-08-04 09:20:39 UTC
Permalink
Post by IV
Post by IV
exp(sin(4 pi)) = 1
4 pi = Arcsin(Ln(1))
4 pi = arcsin(ln(1)) + 4 pi
4 pi = 4 pi
Das war unklar.
Post by IV
exp(sin(4 pi)) = 1
4 pi = Arcsin(Ln(1))
Wo genau taucht denn hier eine Relation auf? Also eine, die keine
Funktion ist? Das sieht mir alles so verdächtig nach
Funktionsschreibweisen aus.
Post by IV
Ich werde also auch den bekannten Satz über die "Zusammensetzung" der
Umkehrrelation 'zusammengesetzter' Relationen mit aufnehmen müssen.
Welchen Satz meinst du?
Post by IV
Und erst jetzt kommen DB und WB ins Spiel, nämlich die der partiellen
Umkehrfunktionen ('Zweige' der Umkehrrelation), und auch nur die, die
überhaupt relevant sind für die Gleichung.
4 pi = arcsin(ln(1)) + 4 pi
4 pi = 4 pi
Warum schreibst du hier zwei Gleichungen untereinander?

Wie lautet denn nun die "partielle Umkehrfunktion"? Bzw. die
"partielle_n_ Umkehrfunktion_en_", die "für diese Gleichung relevant"
sind?

hs
IV
2018-08-04 11:23:40 UTC
Permalink
Post by IV
exp(sin(4 pi)) = 1
4 pi = Arcsin(Ln(1))
4 pi = arcsin(ln(1)) + 4 pi
Wo genau taucht denn hier eine Relation auf? Also eine, die keine Funktion
ist? Das sieht mir alles so verdächtig nach Funktionsschreibweisen aus.
Arcsin und Ln sind bekanntlich Bezeichner für Relationen (hier:
Multifunktionen).
arcsin und ln sind bekanntlich Bezeichner für Funktionen.
Post by IV
4 pi = arcsin(ln(1)) + 4 pi
4 pi = 4 pi
Warum schreibst du hier zwei Gleichungen untereinander?
Das nennt sich Gleichungsumformung.
Wie lautet denn nun die "partielle Umkehrfunktion"? Bzw. die "partielle_n_
Umkehrfunktion_en_", die "für diese Gleichung relevant" sind?
f mit f: z |-> f(z) = exp(z)
g mit g: z |-> sin(z)
die beiden hier relevanten partiellen Umkehrfunktionen:
f^{-1}: {1} --> {ln(1)} , z |-> 0
g^{-1}: {0} --> {arcsin(0) + 4 pi}, z |-> 4 pi
H0Iger SchuIz
2018-08-04 12:21:21 UTC
Permalink
Post by IV
Post by IV
exp(sin(4 pi)) = 1
4 pi = Arcsin(Ln(1))
4 pi = arcsin(ln(1)) + 4 pi
Wo genau taucht denn hier eine Relation auf? Also eine, die keine Funktion
ist? Das sieht mir alles so verdächtig nach Funktionsschreibweisen aus.
Arcsin und Ln sind bekanntlich Bezeichner für Relationen
Soso, bekanntlich. Allerdings verstehe ich nicht, warum denn alles so
geschrieben wird, als seien es Funktionen.
Post by IV
Multifunktionen).
Auch ein schönes Wort. Lieb, dass du das mal nennst.
Post by IV
arcsin und ln sind bekanntlich Bezeichner für Funktionen.
Post by IV
4 pi = arcsin(ln(1)) + 4 pi
4 pi = 4 pi
Warum schreibst du hier zwei Gleichungen untereinander?
Das nennt sich Gleichungsumformung.
Aha. Warum merkt man davon nix? Warum werden keine Umformungsschritte
angegeben? Und welcherlei ist die Beziehung zwischen den Gleichungen?
Sind die äquivalent? Folgt die eine aus der anderen? Gibt es denn für so
etwas keine Schreibweisen?

Das Gleiche könnte ich auch für
Post by IV
Post by IV
4 pi = Arcsin(Ln(1))
4 pi = arcsin(ln(1)) + 4 pi
fragen.

hs
H0Iger SchuIz
2018-08-04 09:20:40 UTC
Permalink
Post by IV
Post by H0Iger SchuIz
Post by IV
Ich habe hier von Euch u. a. gelernt: Erstmal alles Benötigte
mathematisch korrekt definieren, dann jede Aussage mathematisch korrekt
formulieren und beweisen.
Nein. Man kann, wenn überhaupt, nur die wahren Aussagen beweisen.
Dein "Nein" scheint mir zu streng zu sein.
Ist es nicht. _Jede_ Aussage beweisen zu können, ist falsch
Post by IV
Post by H0Iger SchuIz
Und was hat diese Erklärung mit dem Stil von Ritt zu tun?
Erstens: Bei Liouville und Ritt geht Alles ohne explizit genannte
Definitions- und Wertebereiche.
Was heißt "ohne _explizit_ genannte Definitions- und Wertebereiche"?
Werden welche impliziert?

Und was heißt es ginge "Alles"? Unklar.
Post by IV
Zweitens: Erst nachdem ich das Benötigte korrekt definiert, formuliert und
bewiesen habe, kann ich mich auf das spärliche Formulierungsniveau von Ritt
begeben - im Sinne der potentiellen Anwender.
Was genau hat der fiktive Anwender dir denn getan, dass du ihn mit
diesem Niveau ärgern möchtest?
Post by IV
Post by H0Iger SchuIz
Post by IV
Post by H0Iger SchuIz
f: {0} -> {1}, x |-> x + exp(sin(x))
ist offensichtlich bijektiv und hat die Umkehrfunktion
f^-1: {1} -> {0}, x |-> 0
Ich weiß.
Für Dich hatte ich wieder zu schnell formuliert - weil all das noch nicht
zum im Moment zu bearbeitenden Arbeitspaket gehört. Gemeint ist ein
Funktionsterm, der keine Konstante ist.
x + exp(sin(x)) ist eine Konstante? Interessant.
In x |-> 0 ist der Funktionsterm, 0, eine Konstante.
Ah, ok. Dann nehmen wir

f^-1: {1} -> {0}, x |-> 1-x

Besser?
Post by IV
Post by H0Iger SchuIz
Damit meine ich, dass für die Funktion
f: {0} -> {1}, x |-> x + exp(sin(x))
die Gleichung f(x) = 3 keine Lösung hat.
Ich sehe nicht, was Deine Funktion mit der von mir gegebenen Gleichung zu
tun hat.
Meine Funktion ist das schon mal nicht. Das Beispiel ahbe ich nicht
eingeführt.
Post by IV
Implizite Antworten verstehe ich nicht.
Noch expliziter kann ich's nicht hinschreiben. Nachdenken hilft.
Post by IV
Post by H0Iger SchuIz
Post by IV
Post by H0Iger SchuIz
Dann: Wenn ein geschlossener Ausdruck als Funktionsterm existiert, dann
kann die Gleichung durch Umformen aufgelöst werden, wenn nicht, dann
nicht. Oder man wendet meinen Satz 4 an und kriegt die Funktionsterme der
partiellen Umkehrfunktionen geliefert. In diese braucht man dann nur
noch die 3 an Stelle des x einsetzen.
Vorrechnen!
Erst im vorvorletzten Arbeitspaket.
Also kann man die Gleichung nun durch Anwenden des Satzes lösen oder
nicht? Butter bei die Fische.
Meine bisherigen Euch vorliegenden Satz-Entwürfe sagen etwas über globale
und partielle Umkehrfunktionen, nichts über Gleichungen.
Also nein. Mir dünkt hier nimmt jemand den Mund recht voll, wenn es
darum geht zu erzählen, was er so alles beweisen will. Wenn's dann
konkret wird, verschluckt er sich an den großen Brocken.
Post by IV
Meine Gleichung x + exp(sin(x)) = 3?
Wenn nichts weiter über die DB und WB der 'Gliedfunktionen' und der
'Gleichungsfunktion' gegeben ist, muß man die DB und WB von exp und sin
nehmen.
Ach, muss "man" das? Und was sollen denn "DB und WB von exp und sin"
sein, wenn über diese nichts bekannt ist.
Post by IV
Mit Ritts Satz und einem meiner weiteren Satz-Entwürfe folgt,
Aus Entwürfen würde ich keine Schlussfolgerung ziehen wollen.
Post by IV
daß x +
exp(sin(x)) nicht elementar umkehrbar ist. Da wir im Moment keine andere
Funktionenklasse wissen, durch die x + exp(sin(x)) in Form eines
Funktionsterms einer 'unverzweigt verketteten' Funktion darstellbar ist,
Diese Unmöglichkeit der Darstellung wurde in etwa wo gezeigt?
Post by IV
Post by H0Iger SchuIz
Post by IV
Ich bin noch weit entfernt davon, eine v o l l s t ä n d i g e
Lösungstheorie für 'gewöhnliche' Gleichungen anbieten zu können. In der
Literatur habe ich eine solche auch noch nicht gefunden.
Warum bloß?
Weil nicht jeder Liouville, Ritt, Rosenlicht, Khovanskii und Burda kennt.
Ach, daran liegt das.

Aber, erstmal: Was sind denn wohl "gewöhnliche Gleichungen"? Und wie
soll eine "vollständige Lösungstheorie" aussehen.
Post by IV
Post by H0Iger SchuIz
Post by IV
Post by H0Iger SchuIz
Mal sehen: exp(sin(0)) = 1. Aber (Umkehrung): sin(exp(1)) = sin(e) \not=
0. Nee, mit dem Term stimmt etwas nicht.
...
Und was stimmt dann mit meinem Beispiel oben nicht?
Ja, mit dem Term stimmt etwas nicht.
gegeben: die Gleichung exp(sin(0)) = 1
'Umkehrung' durch Gleichungsumformung oder bei komplexerem Funktionsterm
der 'Gleichungsfunktion' einfacher durch direktes Bilden der partiellen
Umkehrfunktionen über meine Sätze, erst mit Hilfe von Ritts Satz und
meinem Satz kann man herleiten, daß das mit elementaren Funktionen bzw.
elementaren Operationen überhaupt möglich ist: 0 = arcsin(ln(1)). 0 = 0.
Ah, das ist aber ein ganz anderer Term als eben. Da haben wir doch wohl
keinen Fehler gemacht?
Wir nicht. Du.
Hört, hört, Jetzt mal Butter bei die Fische. Welchen Fehler habe ich
gemacht? Schrieb ich etwa:

|"Die Umkehrfunktionen der durch den Funktionsterm exp(sin(x))
| repräsentierten bijektiven Funktionen haben den Funktionsterm
| sin(exp(x))"

Oder wer war das?
Post by IV
Die Umkehrung erfolgt (wie beim Gleichunglösen auch) durch
Anwenden der partiellen Umkehrfunktionen der Gliedfunktionen in umgekehrter
Reihenfolge.
Post by H0Iger SchuIz
Wie sieht's denn mit exp(sin(4pi)) = 1 aus. Kann man das auch umkehren?
Also eigentlich ist mein Thema "globale und partielle Umkehrfunktionen
zusammengesetzter Funktionen einer Variablen, und 'gewöhnliche' Gleichungen
einer Variablen".
Und? Dazu hast du ein Beispiel gebracht und ich habe etwas nachgefragt.
Post by IV
Ist die Frage, wonach umgestellt werden soll: nach sin(4pi), 4, oder pi.
Nein, das ist nicht die Frage. Zumindest nicht meine.

Ich frage mal anders. Haben die "Umkehrfunktionen der durch den
Funktionsterm exp(sin(x)) repräsentierten[1] bijektiven Funktionen"
einen (gemeinsamen) Funktionsterm? Oder vielmehr, werden jene
Umkehrfunktionen durch einen (gemeinsamen) Funktionsterm repräsentiert?
Wie lautet dieser?

Und, werden die (offensichtlich bijketiven) Funktionen

f: {0} -> {1}, x |-> exp(sin(x))

und

g: {4pi} -> {1}, x |-> exp(sin(x))

durch den Funktionsterm exp(sin(x)) "repräsentiert"?
Post by IV
Daher weiß man sofort, daß man hier mit elementaren partiellen
Umkehrfunktionen (elementaren Operationen) auskommt.
exp(sin(4 pi)) = 1
4 pi = Arcsin(Ln(1))
4 pi = arcsin(ln(1)) + 4 pi
4 pi = 4 pi
Was passiert hier? Soll hier etwas gerechnet werden? Was sind das für
vier Gleichungen, die da untereinander stehen? Haben die etwas
miteinander zu tun? Unklar.
Post by IV
Post by H0Iger SchuIz
Post by IV
partielle Umkehrfunktionen (Hach, schön, daß wir diesen Terminus
herausarbeiten konnten.)
Konnten "wir" das? Was ist denn nun eine partielle Umkehrfunktion?
Ja, mit Eurer Hilfe habe ich es gefunden.
https://en.wikipedia.org/wiki/Inverse_function#Partial_inverses
Was hast du denn da gefunden? Da wird weder eine Definition partieller
Umkehrfunktionen angegeben, noch irgendwelche Eigenschaften benannt. Es
gibt lediglich ein Beispiel. Könnte das nicht auch als Beispiel für
lokale Umkehrfunktionen dienen? Wo ist da die Abgrenzung?
Post by IV
Post by H0Iger SchuIz
Post by IV
Versuche doch bitte nicht nur, in dem allem keinen Sinn zu sehen,
Ich hinterfrage, nicht mehr, nicht weniger. Wenn dich meine Hinweise nicht
interessieren (oder du sie nicht verstehst), dann ist das eben so. Ich
halte es jedenfalls für gequirlten Quatsch, Funktionen ohne
Definitionsbereich zu betrachten.
Ich betrachte auch nicht Funktionen, sondern
'Gruppen'/'Klassen'/'Mengen'/'Zoos'
Oh, das sind aber viele Begriffe. Schön, dass du die mal so kontextfrei
auflistest. In den von dir formulierten "Sätzen" tauchten die aber nicht
auf. Inwiefern "betrachtest" du die?
Post by IV
von Funktionen mit gleichem
Funktionsterm oder gleicher Form des Funktionsterm.
Nein, das geben die Formulierungen deiner Sätze nicht her. Wenn du so
etwas betrachten möchtest, musst du das auch entsprechend formulieren.
Wenn du das hinkriegst.
Post by IV
Post by H0Iger SchuIz
Ich glaube auch nicht, dass Ritt das im Sinn gehabt hat, nur weil er das
etwas lax aufgeschrieben hat.
Na, er hat immerhin noch geschrieben "die Funktion phi(z)".
Was schon Quatsch ist, weil er da ja Funktion und Funktionswert
verwechselt. Habe ich schon erwähnt, dass mir dieser Stil nicht gefällt?
Mein Tipp: zeitgemäße und exakte Formulierungen verwenden.
Post by IV
Post by H0Iger SchuIz
Prima. Dann ist ja alles in Ordnung. Vergess' nicht, ein paar Beispiele in
deine Veröffentlichungen zu schreiben, die kucke ich mir gerne an.
Mathematische Veröffentlichungen zeichnen sich zu meinem Leidwesen leider
dadurch aus, daß sie selten Beispiele enthalten.
Kommt vielleicht ein Bisschen darauf an, was man so liest. Aber ja,
nicht alle Autoren erkennen die Nützlichkeit von Beispielen für's
Verstehen. Mach's einfach besser!

hs

[1] By the way, wurde schon definiert was durch einen Funktionesterm
repräsentierte Funktionen sind?
IV
2018-08-04 12:17:58 UTC
Permalink
Post by H0Iger SchuIz
daß x + exp(sin(x)) nicht elementar umkehrbar ist. Da wir im Moment keine
andere Funktionenklasse wissen, durch die x + exp(sin(x)) in Form eines
Funktionsterms einer 'unverzweigt verketteten' Funktion darstellbar ist,
Diese Unmöglichkeit der Darstellung wurde in etwa wo gezeigt?
Ritt 1925: Struktursatz für bijektive elementare Funktionen. Für weitere
Funktionenklassen sind keine entsprechenden Struktursätze bekannt. Die will
ja ich erst formulieren - weil's kein anderer macht.
Post by H0Iger SchuIz
Ich frage mal anders. Haben die "Umkehrfunktionen der durch den
Funktionsterm exp(sin(x)) repräsentierten[1] bijektiven Funktionen" einen
(gemeinsamen) Funktionsterm? Oder vielmehr, werden jene Umkehrfunktionen
durch einen (gemeinsamen) Funktionsterm repräsentiert?
Der Funktionsterm der Umkehrfunktion setzt sich aus den "Funktionstermen"
der Umkehrrelationen der Gliedfunktionen zusammen.
Post by H0Iger SchuIz
Wie lautet dieser?
F mit F: x |-> exp(sin(x))
F^{-1} mit F: x-> Arcsin(Ln(x))
Post by H0Iger SchuIz
Post by H0Iger SchuIz
Wie sieht's denn mit exp(sin(4pi)) = 1 aus. Kann man das auch umkehren?
...
exp(sin(4 pi)) = 1
4 pi = Arcsin(Ln(1))
4 pi = arcsin(ln(1)) + 4 pi
4 pi = 4 pi
Was passiert hier? Soll hier etwas gerechnet werden? Was sind das für vier
Gleichungen, die da untereinander stehen? Haben die etwas miteinander zu
tun? Unklar.
Post by H0Iger SchuIz
Post by IV
partielle Umkehrfunktionen (Hach, schön, daß wir diesen Terminus
herausarbeiten konnten.)
Konnten "wir" das? Was ist denn nun eine partielle Umkehrfunktion?
Ja, mit Eurer Hilfe habe ich es gefunden.
https://en.wikipedia.org/wiki/Inverse_function#Partial_inverses
Was hast du denn da gefunden? Da wird weder eine Definition partieller
Umkehrfunktionen angegeben, noch irgendwelche Eigenschaften benannt. Es
gibt lediglich ein Beispiel. Könnte das nicht auch als Beispiel für
lokale Umkehrfunktionen dienen? Wo ist da die Abgrenzung?
"partielle Umkehrfunktion" ist der Obebegriff von "lokale Umkehrfunktion.
Post by H0Iger SchuIz
By the way, wurde schon definiert was durch einen Funktionsterm
repräsentierte Funktionen sind?
Nö, auch das muß ich als Laie wohl selber machen.
Die Menge / Klasse der Funktionen, die durch denselben Funktionsterm
definiert sind.
Gibt's dafür einen mathematischen Begriff?
H0Iger SchuIz
2018-08-04 12:38:56 UTC
Permalink
Post by IV
Post by H0Iger SchuIz
daß x + exp(sin(x)) nicht elementar umkehrbar ist. Da wir im Moment keine
andere Funktionenklasse wissen, durch die x + exp(sin(x)) in Form eines
Funktionsterms einer 'unverzweigt verketteten' Funktion darstellbar ist,
Diese Unmöglichkeit der Darstellung wurde in etwa wo gezeigt?
Ritt 1925: Struktursatz für bijektive elementare Funktionen.
Äh, nein. Der Satz von Ritt gibt an, unter welchen Bedingungen ene
bestimmte Darstellung existiert. Darüber, wann sie nicht existieren
kann, sagt er nichts aus.

Konkret wollte ich aber fragen, wo denn gezeigt wurde, dass Funktionen
mit Funktionsterm x + exp(sin(x)) nicht in dieser Form dargestellt
werden können.
Post by IV
Post by H0Iger SchuIz
Ich frage mal anders. Haben die "Umkehrfunktionen der durch den
Funktionsterm exp(sin(x)) repräsentierten[1] bijektiven Funktionen" einen
(gemeinsamen) Funktionsterm? Oder vielmehr, werden jene Umkehrfunktionen
durch einen (gemeinsamen) Funktionsterm repräsentiert?
Der Funktionsterm der Umkehrfunktion setzt sich aus den "Funktionstermen"
der Umkehrrelationen der Gliedfunktionen zusammen.
Post by H0Iger SchuIz
Wie lautet dieser?
F mit F: x |-> exp(sin(x))
F^{-1} mit F: x-> Arcsin(Ln(x))
Hm, eben waren Arcsin und Ln nur Relationen. Jetzt dann doch Funktionen?
Oder mogeln wir uns hier um etwas 'rum? Und darf ich das so verstehen,
dass der gleiche Term der für alle Umkehfunktionen ergibt. Oder hängt
der womöglich doch von den gewählten Definitions- und Wertebereichen ab?
Post by IV
Post by H0Iger SchuIz
Was hast du denn da gefunden? Da wird weder eine Definition partieller
Umkehrfunktionen angegeben, noch irgendwelche Eigenschaften benannt. Es
gibt lediglich ein Beispiel. Könnte das nicht auch als Beispiel für
lokale Umkehrfunktionen dienen? Wo ist da die Abgrenzung?
"partielle Umkehrfunktion" ist der Obebegriff von "lokale Umkehrfunktion.
Aha. Kann ich nicht beurteilen so lange der Begriff der "partielle
Umkehrfunktion" noch darauf wartet, definiert zu werden. Wenn du das
gemacht hast, kannst du ja auch Beispiele anfügen, die die
Klassifikation als Obebegriff rechtfertigen.
Post by IV
Post by H0Iger SchuIz
By the way, wurde schon definiert was durch einen Funktionsterm
repräsentierte Funktionen sind?
Nö, auch das muß ich als Laie wohl selber machen.
Natürlich musst du das machen. Du willst den Begriff ja auch verwenden.
Patsch!

Gewöhn dich daran, dass deine "Aussagen" keinen Sinn ergeben, so lange
die Begriffe darin nicht sämtlich definiert sind. Wenn du weißt, was es
sein soll, wovon du redest, kannst du auch eine Definition angeben.

hs
H0Iger SchuIz
2018-08-04 13:17:36 UTC
Permalink
Post by H0Iger SchuIz
Post by IV
Der Funktionsterm der Umkehrfunktion setzt sich aus den "Funktionstermen"
der Umkehrrelationen der Gliedfunktionen zusammen.
Post by H0Iger SchuIz
Wie lautet dieser?
F mit F: x |-> exp(sin(x))
F^{-1} mit F: x-> Arcsin(Ln(x))
Hm, eben waren Arcsin und Ln nur Relationen. Jetzt dann doch Funktionen?
Oder mogeln wir uns hier um etwas 'rum? Und darf ich das so verstehen,
dass der gleiche Term der für alle Umkehfunktionen ergibt. Oder hängt
der womöglich doch von den gewählten Definitions- und Wertebereichen ab?
Ich habe hier naoh mal dr[ber geguckt. Ja, hier wird sich um etwas
'rumgemogelt. Ich habe nach dem Funktionsterm der Umkehrfunktionen
gefragt. Als Antwort bekomme ich einen "'Funktionsterm' von
Umkehrrelationen". Hier schreibt er 'Funktionsterm' wohl deshalb mit
Anfüheungsstrichen, weil er weiß, dass der Begriff nicht passt.

Bekanntes Spielche. Erst wird großartug behauptet, wie einfach es doch
ist, mit Hilfe der "Struktursätze" den Term der Umkehrfunktion zu
ermitteln, man bräuchte ja nur die Terme der Umkehrfunktionen der
Gliedfunktionen "zusammenzusetzen". Wenn man dann an einen einfachen
aber konkreten Beispiel nachfragt, kommt da nichts. da wird denn statt
dessen eine Umkehrrelation benannt, die man -- schön Wischiwaschi -- wie
ein Funktion verwendet. Plumps.

Dabei ist es doch nun mal so, dass eben kucken muss, welchen Defitions-
und Wertebereiche es denn nun konkret sind, ob die dann vorliegende
Funktion tatsächlich bijektiv ist und welcher Funktionsterm sich dann
für die Umkehrfunktion ergibt.

Bleiben wir man beim Beispiel. Durch die Periodizität des Sinus, ergibt
sich, dass die Drfinitionsbereich hinreichend klein sein müssen, damit
überhaupt etwas Injektives herauskommt.

f: [0,pi/2] -> [1,e], x |-> exp(sin(x))

ist bijektiv. Die Umkehrfunktion ist

f^-1: [1,e] -> [0,pi/2] , arcsin(ln(x))

Aber:

g: [4pi,(9/2)pi] -> [1,e], x |-> exp(sin(x))

ist ebenfalls bijektiv. Die Umkehrfunktion ist

g^-1: [1,e] -> [4pi,(9/2)pi] , 4pi + arcsin(ln(x))

Andere Definitionsbereiche, anderer Term der Umkehrfunktion. Aber ganz
ohne "Struktursatz" und Umkehrrelationen. Die ganze Idee, man schaue sch
nur die Funktionsterme an, Definitions- und Wertebereiche muss man nicht
betrachten, ist Kappes.

hs
IV
2018-08-04 14:06:59 UTC
Permalink
Post by H0Iger SchuIz
Andere Definitionsbereiche, anderer Term der Umkehrfunktion. Aber ganz
ohne "Struktursatz" und Umkehrrelationen. Die ganze Idee, man schaue nur
die Funktionsterme an, Definitions- und Wertebereiche muss man nicht
betrachten, ist Kappes.
Aufgabenstellung: Die 'gewöhnliche' Gleichung x + exp(sin(x)) = 3 soll
gelöst werden.
Der Laie sieht keine Funktionen, keine Definitions- und Wertebereiche, keine
algebraischen Zahlen. Er rechnet wild drauflos und versucht, die Gleichung
nach x umzustellen. Mit Ritts Satz sieht man sofort, daß die
"Gleichungsfunktion" nicht in der Form ist wie man sie bräuchte, daß die
Gleichung eventuell gar nicht allein durch elementare Operationen (=
elementare Operationen) nach x aufzulösen ist.
Da ich im Moment noch in Ritts Beweis feststecke, kann ich das noch nicht
auf andere Funktionenklassen als die Elementaren Funktionen erweitern. Ich
habe auch noch keine Begriffe, um meine beiden Vermutungen bezüglich anderer
Funktionenklassen in Worte oder Formeln zu fassen.
in Deutsch, nicht in Mathematisch: Ich möchte dem Anwender zeigen: Wenn ein
Funktionsterm F(z) in "linearer"/"unverzweigter" Verkettungsform vorliegt,
dann kann jede der partiellen Umkehrfunktionen der Funktion F, auch die
benötigte, aus Funktionstermen der partiellen Umkehrfunktionen der
"Gliedfunktionen" "zusammengesetzt" werden, bzw. die Gleichung einfach durch
Anwenden der partiellen Umkehrfunktion auf systematische Weise gelöst
werden, ohne wild oder vielleicht sogar sinnlos herumzuprobieren. Wenn die
Gleichung nach x umgestellt ist, dann braucht man nur die 3 in den
"funktionsterm" der Umkehrrelation einsetzen, und alle benötigten
Definitions- und Wertebereiche und damit die jeweils benötigten partiellen
Umkehrfunktionen ergeben sich vollkommen systematisch - praktisch von
selbst.
Aus der Struktur einer gegebenen "zusammengesetzten Funktion" läßt sich, wie
Ritt für die Elementaren Funktionen zeigt, die Funktionenklasse ihrer
partiellen Umkehrfunktionen ablesen: Eine algebraische Funktion hat nun mal
nur algebraische partielle Umkehrfunktionen, exp nur solche die eine
Teilmenge von Ln sind, und ln nur solche die eine Teilmenge von Exp sind.
Die Verallgemeinerung für Funktionenklassen die auch andere
Standardfunktionen enthalten, möchte ich später zeigen.

Aber um all das geht es hier noch nicht. Hier geht es zunächst nur um meine
Sätze 3 und 4 und um die verschiedenen benötigten mathematischen Begriffe.
H0Iger SchuIz
2018-08-05 02:19:20 UTC
Permalink
Post by IV
Post by H0Iger SchuIz
Andere Definitionsbereiche, anderer Term der Umkehrfunktion. Aber ganz
ohne "Struktursatz" und Umkehrrelationen. Die ganze Idee, man schaue nur
die Funktionsterme an, Definitions- und Wertebereiche muss man nicht
betrachten, ist Kappes.
Aufgabenstellung: Die 'gewöhnliche' Gleichung x + exp(sin(x)) = 3 soll
gelöst werden.
Was ist eine "gewöhnliche Gleichung"?
Post by IV
Der Laie sieht keine Funktionen, keine Definitions- und Wertebereiche, keine
algebraischen Zahlen.
Das heißt, er weiß noch nicht mal, was für x eingesetzt werden kann? Was
stellt er dich dann unter einer Lösung vor? Er sucht nach einer Lösung,
weiß aber noch nicht mal wo. So kann es denn also sein, dass er in
München an jeder Haustier klingelt, während die Lösung in Hamburg wohnt.

Ja, nö, is klaa.

"Suche den Apfel?" "Wo?" "Weiß ich doch nicht!".

Wer stellt denn solche Aufgaben? Bzw. in welchen Kontexten sollen die
denn auftreten?
Post by IV
Er rechnet wild drauflos und versucht, die Gleichung
nach x umzustellen. Mit Ritts Satz
Klar, den kennt der Laie.
Post by IV
sieht man sofort, daß die
"Gleichungsfunktion" nicht in der Form ist wie man sie bräuchte, daß die
Gleichung eventuell gar nicht allein durch elementare Operationen (=
elementare Operationen) nach x aufzulösen ist.
Der Satz von Ritt sagt nichts über Auflösbarkeit durch elementare
Operationen.
Post by IV
Da ich im Moment noch in Ritts Beweis feststecke, kann ich das noch nicht
auf andere Funktionenklassen als die Elementaren Funktionen erweitern.
Ich vermute, du kannst noch nicht mal begründen, warum as möglich sein
soll.
Post by IV
Ich
habe auch noch keine Begriffe, um meine beiden Vermutungen bezüglich anderer
Funktionenklassen in Worte oder Formeln zu fassen.
in Deutsch, nicht in Mathematisch: Ich möchte dem Anwender zeigen: Wenn ein
Funktionsterm F(z) in "linearer"/"unverzweigter" Verkettungsform vorliegt,
dann kann jede der partiellen Umkehrfunktionen der Funktion F, auch die
benötigte, aus Funktionstermen der partiellen Umkehrfunktionen der
"Gliedfunktionen" "zusammengesetzt" werden, bzw. die Gleichung einfach durch
Anwenden der partiellen Umkehrfunktion auf systematische Weise gelöst
werden, ohne wild oder vielleicht sogar sinnlos herumzuprobieren. Wenn die
Gleichung nach x umgestellt ist, dann braucht man nur die 3 in den
"funktionsterm" der Umkehrrelation einsetzen, und alle benötigten
Definitions- und Wertebereiche und damit die jeweils benötigten partiellen
Umkehrfunktionen ergeben sich vollkommen systematisch - praktisch von
selbst.
Aha. Ergibt sich von selbst. Woher weiß denn dein zauberverfahren, ob es
Sinn macht reele Zahlen oder endliche Zeichenketten oder geomterische
Objekte oder, oder, in die Funktion einzusetzen. Wie werden denn "von
selbst" diese Informationen generiert, die der fiktive Anwender nicht
hat?

Und du meinst, das Bilden der Umkehrfunktion sei leichter, als die
Gleichung für eine bestimmte Stelle aufzulösen?
Post by IV
Aus der Struktur einer gegebenen "zusammengesetzten Funktion" läßt sich, wie
Ritt für die Elementaren Funktionen zeigt, die Funktionenklasse ihrer
Ritts Satz sagt nichts über partielle Umkehrfunktionen. Er macht eine
Aussage über bijekte Funktionen. Die _Voraussetzung_ in Ritts Satz ist,
dass die betrachtete bijektive Funktion und ihre Umkehrfunktion in einer
bestimmten Klasse liegen.

Interessiert es den fiktiven Anwender eigentlich ganz dringend, in
welcher "Funktionenklasse" die Umkehrfunktion liegt? Suchte er nicht
nach einer Lösung für eine Gleichung?
Post by IV
Eine algebraische Funktion hat nun mal
nur algebraische partielle Umkehrfunktionen, exp nur solche die eine
Teilmenge von Ln sind, und ln nur solche die eine Teilmenge von Exp sind.
Mag ja sein. Aber die spannende Frage ist doch, welche Teilmenge es ist.
Post by IV
Die Verallgemeinerung für Funktionenklassen die auch andere
Standardfunktionen enthalten, möchte ich später zeigen.
Soso.
Post by IV
Aber um all das geht es hier noch nicht. Hier geht es zunächst nur um meine
Sätze 3 und 4 und um die verschiedenen benötigten mathematischen Begriffe.
Jo, dann schreib doch das mal auf. Dann sehen wir weiter.

hs
IV
2018-08-05 10:37:13 UTC
Permalink
Post by H0Iger SchuIz
Post by IV
Die 'gewöhnliche' Gleichung
Was ist eine "gewöhnliche Gleichung"?
in Deutsch, nicht in Mathematisch:
Differentialgleichungen, Differenzengleichungen, Funktionalgleichungen und
Rekursionsgleichungen sind Gleichungen. Wie nennt man Gleichungen, die weder
Differentialgleichungen noch Differenzengleichungen, Funktionalgleichungen
oder Rekursionsgleichungen sind?
Post by H0Iger SchuIz
Post by IV
Der Laie ...
Er rechnet wild drauflos und versucht, die Gleichung nach x umzustellen.
Mit Ritts Satz
Klar, den kennt der Laie.
Wenn er ihn kennen würde, bräuchte er gar nicht erst versuchen, die
Kepler-Gleichung allein mit elementaren Operationen umzuformen.
Ich möchte Ritts Satz bekannter machen.
Post by H0Iger SchuIz
Post by IV
sieht man sofort, daß die "Gleichungsfunktion" nicht in der Form ist wie
man sie bräuchte, daß die Gleichung eventuell gar nicht allein durch
elementare Operationen (= elementare Funktionen) nach x aufzulösen ist.
Der Satz von Ritt sagt nichts über Auflösbarkeit durch elementare
Operationen.
Das sagen ein paar von mir noch zu formulierende Sätze.

Die gehen in etwa so:

Ist F(x) = 0 eine Gleichung, x deren Lösungsvariable, und F eine Funktion,
dann kann die Gleichung durch Anwendung partieller Umkehrfunktionen von F
nach x umgestellt werden.

Ist F eine surjektive Funktion, dann kann die Umkehrrelation von F
vollständig in partielle Umkehrfunktionen von F 'zerlegt' werden.

Ist F eine surjektive Funktion, dann ist F auf jeder Teilmenge des
Definitionsbereichs von F durch partielle Umkehrfunktionen von F
'umkehrbar'.

Sei F(x) = 0 eine Gleichung, und x deren Lösungsvariable.
Ist F eine elementare Funktion, für die keine "lineare"
Verkettungsdarstellung (die aus Ritts Satz, wo alle algebraischen Funktionen
einstellige Funktionen sind) existiert, dann kann die Gleichung nicht allein
durch elementare Operationen nach x umgestellt werden.
Post by H0Iger SchuIz
Post by IV
Da ich im Moment noch in Ritts Beweis feststecke, kann ich das noch nicht
auf andere Funktionenklassen als die Elementaren Funktionen erweitern.
Ich vermute, du kannst noch nicht mal begründen, warum das möglich sein
soll.
Weil das Gemeinsame zwischen den Elementaren Funktionen, den Liouvilleschen
Funktionen und anderen Funktionenklassen, die durch einen Körper, der nur
einstellige Standardfunktionen und deren Umkehrfunktionen enthält, erzeugt
werden, die algebraischen Funktionen sind.
Sei F eine Funktion aus einer so charakterisierten Funktionenklasse. Die
Funktion F ist genau dann innerhalb dieser Funktionenklasse 'umkehrbar',
wenn für sie eine 'lineare' 'Verkettungsdarstellung' existiert, also eine
'Verkettungsdarstellung', deren algebraische Funktionen sämtlich einstellig
sind.
Post by H0Iger SchuIz
Post by IV
Wenn die Gleichung nach x umgestellt ist, dann braucht man nur die 3 in
den "Funktionsterm" der Umkehrrelation einsetzen, und alle benötigten
Definitions- und Wertebereiche und damit die jeweils benötigten
partiellen Umkehrfunktionen ergeben sich vollkommen systematisch -
praktisch von selbst.
Aha. Ergibt sich von selbst. Woher weiß denn dein zauberverfahren, ob es
Sinn macht reele Zahlen oder endliche Zeichenketten oder geometrische
Objekte oder, oder, in die Funktion einzusetzen. Wie werden denn "von
selbst" diese Informationen generiert, die der fiktive Anwender nicht hat?
Man setzt die gegebene 3 in den "Funktionsterm" der Umkehrrelation ein und
kann so alle Lösungen der Gleichung auf systematische Weise berechnen.
Natürlich kann man sich auch Gedanken über den DB der Gleichung machen und
über DB und WB der partiellen Umkehrfunktionen entscheiden, welche Lösungen
man nicht berechnen will. Man braucht das aber nur für die tatsächlich nach
Einsetzen der 3 benötigten partiellen Umkehrfunktionen machen, nicht für die
nicht relevanten DB und WB.
Post by H0Iger SchuIz
Und du meinst, das Bilden der Umkehrfunktion sei leichter, als die
Gleichung für eine bestimmte Stelle aufzulösen?
Hier geht es um Mathematik. Ich schaue wie weit ich mit meinem Ansatz
komme - nur um der Mathematik willen. Mögliche Anwendungen können später
gesucht werden oder auch von Anderen entwickelt werden.
Auch Ritts Satz ist vollkommen unbekannt und scheint mir in der Literatur
noch nie eingesetzt worden zu sein.
Post by H0Iger SchuIz
Post by IV
Aus der Struktur einer gegebenen "zusammengesetzten Funktion" läßt sich,
wie Ritt für die Elementaren Funktionen zeigt, die Funktionenklasse ihrer
partiellen Umkehrfunktionen ablesen
Ritts Satz sagt nichts über partielle Umkehrfunktionen. Er macht eine
Aussage über bijekte Funktionen. Die _Voraussetzung_ in Ritts Satz ist,
dass die betrachtete bijektive Funktion und ihre Umkehrfunktion in einer
bestimmten Klasse liegen.
Es ist eine triviale Folgerung aus Ritts Satz.
Post by H0Iger SchuIz
Interessiert es den fiktiven Anwender eigentlich ganz dringend, in welcher
"Funktionenklasse" die Umkehrfunktion liegt? Suchte er nicht nach einer
Lösung für eine Gleichung?
Er möchte wissen, ob die Gleichung "symbolisch" gelöst werden kann oder nur
numerisch. Dazu muß er die "Monome" der "Gleichungsfunktion" heraussuchen.
Wenn die "Monome" alle "linear verkettet" vorliegen, dann kann die Gleichung
durch Anwenden partieller Umkehrfunktionen der beteiligten "Monome" nach der
Lösungsvariablen umgestellt werden. Alle partiellen Umkehrfunktionen der
"Gleichungsfunktion" gehören zum Körper, der aus den "Monomen" und ihren
Umkehrrelationen gebildet wird.
Post by H0Iger SchuIz
Post by IV
Eine algebraische Funktion hat nun mal nur algebraische partielle
Umkehrfunktionen, exp nur solche die eine Teilmenge von Ln sind, und ln
nur solche die eine Teilmenge von Exp sind.
Mag ja sein. Aber die spannende Frage ist doch, welche Teilmenge es ist.
Mag sein.
Die allererste Frage aber ist, ob die gegebene Gleichung "Symbolisch" gelöst
werden kann, und wenn ja, in welcher Funktionenklasse, oder ob sie nur
numerisch zu auflösen ist.
(Ich mache einen Unterschied zwischen Auflösen (= nach der Lösungsvariablen
umstellen) und Lösen (= die Lösung angeben). Siehe T. Chow: What is a
Closed-Form Number?)
H0Iger SchuIz
2018-08-05 11:52:12 UTC
Permalink
Post by IV
Hier geht es um Mathematik.
Gut, dass das mal gesagt wird. Man werkt's nämlich auf Anhieb nicht.
Post by IV
Ich schaue wie weit ich mit meinem Ansatz
komme - nur um der Mathematik willen. Mögliche Anwendungen können später
gesucht werden oder auch von Anderen entwickelt werden.
Hm, ja, klar. Erzählst du uns nicht den ganzen Tag, dass due den
Anwender im Blick habest, dass der Anwender dies und jenes tue und man
deshalb so und so formulieren müsse. Und auf einmal (wenn man nämlich
nachfragt), sind die Anwendungen egal, Zunkunftsmusik oder ein PAL[1]?

Das ist der eine Widerspruch. der andere ist, dass, Mathematik um der
Mathematik willen zu betreiben, nicht mit "Ich bin kein Mathematiker und
will auch keiner werden." zusammenpasst.

Mir sind aber auch deine Beweggründe reichlich wurscht. Ich bin ja keine
Therapeut (und will auch keiner werden).

Es sei noch angemerkt, dass der Stil beim Schreiben von Projektanträgen
mit den vielen Buzzwords und tollen Ideen und fast fertigen Ergebnissen
beim Bearbeiten der Projekte durchaus hinderlich sich sein kann. Wenn
man sich in sein Bewerbungsgeschwurbel tief genug verstrickt hat, weiß
man nicht mehr, was man weiß.

hs
IV
2018-08-05 13:05:40 UTC
Permalink
Hm, ja, klar. Erzählst du uns nicht den ganzen Tag, ...
Das ist der eine Widerspruch ...
Wenn man sich in sein Bewerbungsgeschwurbel tief genug verstrickt hat, ...
Ein Endergebnis bekommt man leider immer erst am Ende, nicht zwischendurch,
und schon gar nicht auf ständiges vorzeitiges Nachfragen. Da bekommt man nur
Geschwurbel. Ich mag kein Geschwurbel.
H0Iger SchuIz
2018-08-05 15:57:52 UTC
Permalink
Post by IV
Hm, ja, klar. Erzählst du uns nicht den ganzen Tag, ...
Das ist der eine Widerspruch ...
Wenn man sich in sein Bewerbungsgeschwurbel tief genug verstrickt hat, ...
Ein Endergebnis bekommt man leider immer erst am Ende, nicht zwischendurch,
und schon gar nicht auf ständiges vorzeitiges Nachfragen.
Hat jemand nach einem Endergebnis gefragt? Wäre ja blöd, da du ganz
offensichtlich noch überhaupt keine Ergebnisse präsentieren ganz. Noch
nicht mal kleinste Zwischenergebnisse. Aber lassen wir uns nicht
ablenken. Die von mir aufgezeigten Widersprüche in deinen Aussagen
bezogen sich nicht auf die Frage, ob schon ein Endergebnis vorliege. Im
Gegenteil ging es um den grundsätzlichen Anspruch an dein Projekt.
Post by IV
Da bekommt man nur
Geschwurbel. Ich mag kein Geschwurbel.
Dafür beherrschst du es aber außergewöhnlich gut.

hs

PS: Ich habe wohl eine Fußnote vergessen, sorry.

PAL: Problem anderer Leute
IV
2018-08-04 13:21:04 UTC
Permalink
Post by IV
Post by H0Iger SchuIz
Post by IV
Mit Ritts Satz und einem meiner weiteren Satz-Entwürfe folgt, daß x +
exp(sin(x)) nicht elementar umkehrbar ist. Da wir im Moment keine
andere Funktionenklasse wissen, durch die x + exp(sin(x)) in Form eines
Funktionsterms einer 'unverzweigt verketteten' Funktion darstellbar ist,
Diese Unmöglichkeit der Darstellung wurde in etwa wo gezeigt?
Ritt 1925: Struktursatz für bijektive elementare Funktionen.
Äh, nein. Der Satz von Ritt gibt an, unter welchen Bedingungen eine
bestimmte Darstellung existiert. Darüber, wann sie nicht existieren kann,
sagt er nichts aus.
in Deutsch, nicht in Mathematisch: Er besagt, daß für eine bijektive
elementar umkehrbare Funktion ein Funktionsterm existiert, der eine solche
im Satz angegebene "lineare" / "unverzweigte" Verkettung ist. Daraus folgt,
daß eine elementare Funktion, für die keine solche Darstellung existiert,
nicht elementar umkehrbar ist.
Konkret wollte ich aber fragen, wo denn gezeigt wurde, dass Funktionen mit
Funktionsterm x + exp(sin(x)) nicht in dieser Form dargestellt werden
können.
in Deutsch, nicht in Mathematisch:
Nirgends. Das ist alles noch Zukunftsmusik und gehört nicht in das im Moment
zu bearbeitende Arbeitspaket.
Der Funktionsterm x + exp(sin(x)) ist nicht in der "linearen" /
"unverzweigten" Verkettungs-Form aus Ritts Satz: über offenem(?)
Definitionsbereich sind die Funktionen mit x |-> x und x |-> exp(x)
algebraisch unabhängig. Meine Vermutung ist, daß eine elementare Funktion
über offenem(?) Definitionsbereich mit algebraisch unabhängiger
Argumentliste* keine "lineare" / "unverzweigte" Verkettungs-Darstellung wie
in Ritts Satz haben kann.

*) Argumentliste bzw. Argument(liste):
http://www.mathematik.uni-dortmund.de/lsvii/Preprints/mengen.pdf
http://www.mathi.uni-heidelberg.de/~busam/Analysis.1.2002.SS/MengenSkript.ps
Gewöhn dich daran, dass deine "Aussagen" keinen Sinn ergeben, so lange die
Begriffe darin nicht sämtlich definiert sind. Wenn du weißt, was es sein
soll, wovon du redest, kannst du auch eine Definition angeben.
Erstmal habe ich nur Ideen von Begriffen. Und ich frage hier in Deutsch nach
mathematischen Begriffen, die meinen deutschsprachigen Begriffen entsprechen
könnten. Wenn ich passende mathematische Begriffe genannt bekommen oder
gefunden habe, dann schaue ich mir deren Definitionen an. Für die Definition
vieler meiner Begriffe werden wieder andere Begriffe und deren Definitionen
und deren Begriffe und deren Definitionen und und und .... benötigt.
B***@outlook.de
2018-07-30 19:30:16 UTC
Permalink
Post by IV
Könntet Ihr mir bitte Definitionen oder deren Quellen für folgende
Begriffe nennen? Ich als Laie bin doch dazu kaum in der Lage.
Es geht um Funktionen, die über Funktionsterme definiert sind (closed-form
functions).
Ich fange einfach mal an.
zusammengesetzte Funktion := eine Funktion, deren Funktionsterm aus
mindestens zwei Funktionstermen zusammengesetzt ist.
Funktion F mit F(z) = f(f1(z))
Funktion F mit F(z) = f(f1(f2(z)))
Funktion F mit F(z) = f( f1(z), f2(z) )
Funktion F mit F(z) = f( f1( f11(z), f12(z) ), f2( f21(z)), f3(z) )
Oder muß ich immer schreiben "Funktion F mit z |-> F(z) = ..."?
Diese Definition von "zusammengesetzter Funktion" ist in sich
Widersprüchlich.

GEGENBEISPIEL:

Du wirst mir doch sicher Zustimmen, dass die Funktion

f: IR -> IR, x |-> x

Eine Funktion ist, die über einen Funktionsterm definiert ist.

Die Funktion

g: IR -> {0, 1}, f(x) = 1, falls x rational ist, f(x) = 0, falls x irrational ist

ist keine über einen geschlossenen Funktionsterm definierte Funktion. Ob sie
überhaupt einen geschlossenen Term erlaubt, ist mir nicht bekannt.


Nun ist die Funktion

F: IR -> {0, 1}, F(x) = g(f(x)) aber eine Funktion, die nach deiner Definition
Eine zusammengesetzte Funktion, da sie aus mindestens zwei Funktionstermen, nämlich f unf g, zusammengestzt ist.

Gleichzeitig ist aber auch F(x) = g(x), d.h. sie ist eine nicht-zusammengesetzte

Funktion.

Damit kann mit deiner Definition nicht entschieden werde, ob ein und dieselbe
Funktion nun zusammengesetzt ist oder nicht.


Grosse Frage: Wenn es dir doch augenscheinlich nur auf die Funktionsterme
ankommt, warum hast du deine Theorie nicht auf dem Begriff des Terms aufgebaut?
Der braucht wenigstens nicht einen Definitionsbereich und Wertebereich wie Eine Funktion.
Ausserdem ist genau festgelegt, wie ein Term aussehen muss, so dass man sicher
auch den Begriff "zusammengesetzter Term" widerspruchsfrei definieren kann.
B***@outlook.de
2018-07-30 19:39:21 UTC
Permalink
Post by B***@outlook.de
Post by IV
Könntet Ihr mir bitte Definitionen oder deren Quellen für folgende
Begriffe nennen? Ich als Laie bin doch dazu kaum in der Lage.
Es geht um Funktionen, die über Funktionsterme definiert sind (closed-form
functions).
Ich fange einfach mal an.
zusammengesetzte Funktion := eine Funktion, deren Funktionsterm aus
mindestens zwei Funktionstermen zusammengesetzt ist.
Funktion F mit F(z) = f(f1(z))
Funktion F mit F(z) = f(f1(f2(z)))
Funktion F mit F(z) = f( f1(z), f2(z) )
Funktion F mit F(z) = f( f1( f11(z), f12(z) ), f2( f21(z)), f3(z) )
Oder muß ich immer schreiben "Funktion F mit z |-> F(z) = ..."?
Diese Definition von "zusammengesetzter Funktion" ist in sich
widersprüchlich.
Du wirst mir doch sicher Zustimmen, dass die Funktion
f: IR -> IR, x |-> x
eine Funktion ist, die über einen Funktionsterm definiert ist.
Die Funktion
g: IR -> {0, 1}, f(x) = 1, falls x rational ist, f(x) = 0, falls x irrational ist
KORREKTUR: soll natürlich heißen
g: IR -> {0, 1}, g(x) = 1, falls x rational ist, g(x) = 0, falls x irrational ist
Post by B***@outlook.de
ist keine über einen geschlossenen Funktionsterm definierte Funktion. Ob sie
überhaupt einen geschlossenen Term erlaubt, ist mir nicht bekannt.
Nun ist die Funktion
F: IR -> {0, 1}, F(x) = g(f(x)) aber eine Funktion, die nach deiner Definition
Eine zusammengesetzte Funktion, da sie aus mindestens zwei Funktionstermen,
nämlich f und g, zusammengestzt ist.
Gleichzeitig ist aber auch F(x) = g(x), d.h. sie ist eine
nicht-zusammengesetzte Funktion.
ZUSATZ:
… und zwar nicht, weil nur ein einzelner Funktionsterm auf der rechten Seite
steht, sondern weil F genauso wie g keine geschlossene Form ist.
Post by B***@outlook.de
Damit kann mit deiner Definition nicht entschieden werde, ob ein und dieselbe
Funktion nun zusammengesetzt ist oder nicht.
Grosse Frage: Wenn es dir doch augenscheinlich nur auf die Funktionsterme
ankommt, warum hast du deine Theorie nicht auf dem Begriff des Terms aufgebaut?
Der braucht wenigstens nicht einen Definitionsbereich und Wertebereich wie
eine Funktion.
Ausserdem ist genau festgelegt, wie ein Term aussehen muss, so dass man sicher
auch den Begriff "zusammengesetzter Term" widerspruchsfrei definieren kann.
IV
2018-07-30 22:26:42 UTC
Permalink
Post by B***@outlook.de
Post by B***@outlook.de
Gleichzeitig ist aber auch F(x) = g(x), d.h. sie ist eine
nicht-zusammengesetzte Funktion.
… und zwar nicht, weil nur ein einzelner Funktionsterm auf der rechten
Seite steht, sondern weil F genauso wie g keine geschlossene Form ist.
Geschlossene Form: geschlossene Differentialform, geschlossene Formel, oder
geschlossener Ausdruck? (alles Begriffe aus Wikipedia)
H0Iger SchuIz
2018-08-04 09:20:40 UTC
Permalink
Post by IV
Geschlossene Form: geschlossene Differentialform, geschlossene Formel, oder
geschlossener Ausdruck? (alles Begriffe aus Wikipedia)
So, dann sind diese Buchstabenfolgen also benannt. Und jetzt?

hs
IV
2018-08-04 13:32:22 UTC
Permalink
Post by IV
Geschlossene Form: geschlossene Differentialform, geschlossene Formel,
oder geschlossener Ausdruck? (alles Begriffe aus Wikipedia)
So, dann sind diese Buchstabenfolgen also benannt. Und jetzt?
https://en.wikipedia.org/wiki/Closed-form_expression
Es gibt keine exakte Definition dafür. Die muß ich als Laie auch erst selber
herausarbeiten.
Meine ersten Versuche dazu:
https://math.stackexchange.com/questions/175469/proving-that-special-functions-do-not-have-closed-form-expression/2503259#2503259
https://math.stackexchange.com/questions/2258520/why-some-inverse-functions-do-not-have-a-closed-form/2368212#2368212
IV
2018-07-30 22:22:32 UTC
Permalink
Post by B***@outlook.de
Post by IV
zusammengesetzte Funktion := eine Funktion, deren Funktionsterm aus
mindestens zwei Funktionstermen zusammengesetzt ist.
Funktion F mit F(z) = f(f1(z))
Funktion F mit F(z) = f(f1(f2(z)))
Funktion F mit F(z) = f( f1(z), f2(z) )
Funktion F mit F(z) = f( f1( f11(z), f12(z) ), f2( f21(z)), f3(z) )
Oder muß ich immer schreiben "Funktion F mit z |-> F(z) = ..."?
Diese Definition von "zusammengesetzter Funktion" ist in sich
widersprüchlich.
Du wirst mir doch sicher zustimmen, dass die Funktion
f: IR -> IR, x |-> x
eine Funktion ist, die über einen Funktionsterm definiert ist.
Die Funktion
g: IR -> {0, 1}, f(x) = 1, falls x rational ist, f(x) = 0, falls x irrational ist
ist keine über einen geschlossenen Funktionsterm definierte Funktion. Ob
sie überhaupt einen geschlossenen Term erlaubt, ist mir nicht bekannt.
Ich würde mal sagen: Es ist eine stückweise definierte Funktion, der
Funktionsterm ist definiert, und man kann den Funktionsterm 'umkehren' also
die Funktionsterme der partiellen Umkehrfunktionen bestimmen.
Post by B***@outlook.de
Nun ist die Funktion
F: IR -> {0, 1}, F(x) = g(f(x)) aber eine Funktion, die nach deiner
Definition eine zusammengesetzte Funktion ist, da sie aus mindestens zwei
Funktionstermen, nämlich f und g, zusammengesetzt ist.
Gleichzeitig ist aber auch F(x) = g(x), d.h. sie ist eine
nicht-zusammengesetzte Funktion.
Damit kann mit deiner Definition nicht entschieden werden, ob ein und
dieselbe Funktion nun zusammengesetzt ist oder nicht.
Dasselbe tritt auf bei Funktionen x |-> exp(ln(x)). Ob man eine Funktion als
zusammengesetzte Funktion betrachtet oder nicht, hängt von ihrem
Funktionsterm ab. Eine Funktion kann mehrere Funktionsterme haben. Jede
Funktion mit Funktionsterm kann als zusammengesetzte Funktion dargestellt
werden. Eigentlich geht es bei "zusammengesetzt oder nicht" nicht um die
Funktionen, sondern um Funktionsterme.
Trotzdem wird der Begriff "zusammengesetzte Funktion" sehr häufig verwendet.
Eine Funktion mit Funktionsterm wird über den Funktionsterm definiert. Wenn
dieser 'zusammengesetzt' ist, kann man die Funktion als zusammengesetzte
Funktion behandeln. Wenn dieselbe Funktion neben 'zusammengesetzten'
Funktionstermen auch über 'nicht zusammengesetzte' Funktionsterme definiert
wird, nur dann kann man nicht entscheiden, ob die Funktion zusammengesetzt
ist oder nicht zusammengesetzt. Es ist unerheblich, ob die Funktion auch als
nicht zusammengesetzte Funktion existiert - es interessiert hier nur, wie
sie gerade dargestellt und betrachtet wird.
Post by B***@outlook.de
Grosse Frage: Wenn es dir doch augenscheinlich nur auf die Funktionsterme
ankommt, warum hast du deine Theorie nicht auf dem Begriff des Terms aufgebaut?
Der braucht wenigstens nicht einen Definitionsbereich und Wertebereich wie Eine Funktion.
Ausserdem ist genau festgelegt, wie ein Term aussehen muss, so dass man
sicher auch den Begriff "zusammengesetzter Term" widerspruchsfrei
definieren kann.
Das wurde mir ja erst hinterher klar. Ritt vermengt in seinem Satz ja auch
die Begriffe Funktion und Funktionsterm. Die Verlockung zu den Begriffen
"Komposition" und "zusammengesetzte Funktion" war doch zu verlockend.
Vermutlich werde ich die Sätze einmal für die Funktionsterme und einmal für
die Funktionen formulieren.
Hans Crauel
2018-07-31 20:41:17 UTC
Permalink
IV schrieb
Post by IV
Es geht um Funktionen, die über Funktionsterme definiert sind (closed-form
functions).
Was auch immer das heissen soll. Aber macht nichts. Das kriegen
wir schon hin.
Post by IV
Ich fange einfach mal an.
zusammengesetzte Funktion := eine Funktion, deren Funktionsterm aus
mindestens zwei Funktionstermen zusammengesetzt ist.
Ich mach dann einfach mal weiter.

Lemma I: Jede Funktion, die auf einer Menge mit wenigstens zwei
Elementen definiert ist, ist eine zusammengesetzte Funktion.

Beweis: Es sei f eine Funktion, und es sei F eine echte,
nichtleere Teilmenge des Definitionsbereichs von f sowie
G ihr Komplement. Dann ist f = 1_F f + 1_G f, wobei 1_C
die Indikatorfunktion einer Menge C ist. Mithin ist f eine
zusammengesetzte Funktion.

Lemma II: Jede Funktion, deren Wertebereich ein von {0}
verschiedener Vektorraum ist, ist eine zusammengesetzte Funktion.

Beweis: Ist f eine Funktion, so ist f(x) = f1(x) + f2(x) fuer
alle x aus dem Definitionsbereich von f, wobei f1 = f/3 und
f2 = 2/3 f ist. Mithin ist f eine zusammengesetzte Funktion.

Korollar: 1) Jede Funktion, die die Voraussetzungen des Lemmas
erfuellt, hat einen Funktionsterm.
2) Alle Ergebnisse, die fuer zusammengesetzte Funktionen gelten,
gelten fuer alle Funktionen.

Bemerkung: Die Schreibweise 1_F f + 1_G f ist wohldefiniert,
wenn auf dem Wertebereich von f eine Addition definiert ist.
Andernfalls ist sie als

/
| f(x) fuer x in F
(1_F f + 1_G f)(x) = <
| f(x) fuer x in G
\
zu verstehen.
Sind ueberfluessig, nachdem nunmehr jede Funktion als
zusammengesetzt erkannt worden ist.
Post by IV
univariate Funktion := Funktion einer Variablen
multivariate Funktion := Funktion mehrerer Variabler
Muß ich die Begriffe "Funktion einer Variablen" und "Funktion mehrerer
Variabler" noch definieren?
Ist schon klar: F : C(R,R) -> R, F(f) = f(1) fuer f in C(R,R)
ist eine (reellwertige) Funktion einer Variablen, naemlich von
der Variablen f (wobei C(R,R) fuer die Menge der stetigen
Funktionen von R nach R steht).

Ebenso ist auch f : R^d -> R eine (reellwertige) Funktion einer
Variablen, x mapsto f(x) fuer x in R^d.

Entsprechend ist auch D : C^1(R,R) -> C(R,R), f -> f' (die
Ableitung) eine (C(R,R)-wertige) univariate Funktion; C^1(R,R)
die Menge der stetig differenzierbaren Funktionen von R nach R.

Dagegen ist f : [0,1] x [0,1] -> R, f(x,y) = x, eine (reellwertige)
Funktion zweier Variabler und damit eine multivariate Funktion.
Post by IV
Wie kann man die Definitionen und Begriffsbenennungen verbessern?
Nee, die sind doch prima.
Post by IV
Das Projekt soll der Allgemeinheit dienen, nicht mir persönlich.
Das wird o.B.d.A. nur schwer moeglich sein.

Hans
IV
2018-08-02 20:16:04 UTC
Permalink
Post by IV
Es geht um Funktionen, die über Funktionsterme definiert sind
(closed-form functions).
Was auch immer das heissen soll. Aber macht nichts. Das kriegen wir schon
hin.
Den Begriff Funktionsterm brauche ich nicht extra definieren.
Und nicht jede Funktion wird über einen Funktionsterm definiert.
Post by IV
zusammengesetzte Funktion := eine Funktion, deren Funktionsterm aus
mindestens zwei Funktionstermen zusammengesetzt ist.
Lemma I: Jede Funktion, die auf einer Menge mit wenigstens zwei Elementen
definiert ist, ist eine zusammengesetzte Funktion.
...
Lemma II: Jede Funktion, deren Wertebereich ein von {0} verschiedener
Vektorraum ist, ist eine zusammengesetzte Funktion.
...
Korollar: 1) Jede Funktion, die die Voraussetzungen des Lemmas erfuellt,
hat einen Funktionsterm.
2) Alle Ergebnisse, die fuer zusammengesetzte Funktionen gelten, gelten
fuer alle Funktionen.
Sind ueberfluessig, nachdem nunmehr jede Funktion als zusammengesetzt
erkannt worden ist.
Post by IV
univariate Funktion := Funktion einer Variablen
multivariate Funktion := Funktion mehrerer Variabler
Muß ich die Begriffe "Funktion einer Variablen" und "Funktion mehrerer
Variabler" noch definieren?
Ist schon klar: ...
Post by IV
Wie kann man die Definitionen und Begriffsbenennungen verbessern?
Nee, die sind doch prima.
Wirklich? Oder meinst Du das ironisch? Ich verstehe es wie immer nicht.
Hans CraueI
2018-08-03 09:27:04 UTC
Permalink
IV schrieb
Post by IV
Den Begriff Funktionsterm brauche ich nicht extra definieren.
Jeden mathematischen Begriff muss du extra definieren,
ansonsten gibt es nur Geschwafel.
Post by IV
Und nicht jede Funktion wird über einen Funktionsterm definiert.
Jede Funktion f laesst sich in der Form f = f1 + f2 darstellen
(ausgenommen Funktionen von einer einelementigen in eine
einelementige Menge). Ist in dem von dir komplett geloeschten
Teil ausgefuehrt.
Warum soll das dann kein Funktionsterm sein? Was genau ist
denn dann ein Funktionsterm - ah, das hatten wir doch gerade.
Post by IV
Wirklich? Oder meinst Du das ironisch? Ich verstehe es wie immer nicht.
Du loeschst einfach komplett alle inhaltlichen Punkte.
Soll *das* ironisch sein?

Der Klarheit halber:
Mathematik ist nicht das Aufeinanderhaeufen von Formeln und
Begriffen, sondern ganz vorrangig das aktive (Nach-) Vollziehen
und ggf. auch Erlaeutern von Denkvorgaengen anderen gegenueber.
Letzteres hilft sehr, gerade auch um die eigenen Ueberlegungen
auf Fehlerfreiheit zu ueberpruefen.
Ein Ausweichen auf "Ich bin kein Mathematiker, also kann ich
das nicht und man kann es auch von mir nicht erwarten" wird dem
nicht gerecht. Mathematik ist Jagdtaetigkeit, keine
Sammeltaetigkeit. Anhaeufen von Aussagen ist keine Mathematik.

Definitionen ergeben sich fast von selbst, wenn man einen
Gegenstand mal zumindest halbwegs verstanden hat.

Hans
Carlo XYZ
2018-08-06 14:57:25 UTC
Permalink
Post by Hans CraueI
Jede Funktion f laesst sich in der Form f = f1 + f2 darstellen
(ausgenommen Funktionen von einer einelementigen in eine
einelementige Menge).
Für f: L->R geht id_L o f = f = f o id_R allgemein.
Post by Hans CraueI
Definitionen ergeben sich fast von selbst, wenn man einen
Gegenstand mal zumindest halbwegs verstanden hat.
Wie man's nimmt. Manchmal kann es ganz schön schwer sein,
einen Begriff zu präzisieren, der leistet, was man möchte.

S.a. 2:43 in


Loading...